Download as pdf or txt
Download as pdf or txt
You are on page 1of 148

DIRM

MODULE I

PRINCIPLES AND PRACTICE OF INSURANCE


Supplementary Study Material

The Institute of Chartered Accountants of India


New Delhi

Committee on Insurance and Pension

Released by

CHAPTER 1 INTRODUCTION TO RISK


SECTION - A
1. The degree or level of risk is related to the possibility of occurrence. a. When the possibility of occurrence of an event is higher, then the degree of risk is higher. b. When the possibility of occurrence of an event is lower, then the degree of risk is higher c. both A and B d. none of the above 2. Peril means a. Danger b. Risk c. Cause of loss d. Uncertainty 3. Risk means a. Fear of loss b. A cause of loss c. Measurable Uncertainty d. Unpredictability 4. Life insurance covers Human life risk exposures of a. Untimely Death b. Suicide c. Unemployment d. Divorce 5. A Mortality Table shows a. Number of births b. Incidence of death at specified ages c. Number of deaths d. Both number of births and deaths.

6.

Which one of the following is not a Peril? a) Theft b) Fire c) Burglary

d) Wet Road 7. A Hazard a) Reduces the chances of loss b) Creates the chances of loss c) Increases the frequency and severity of loss d) B and C Organisations are mainly concerned with managing: a) Pure risks b) Speculative risks c) Personal risks d) None of the above Insurance is a risk management technique involving: a) Risk retention b) Risk avoidance c) Loss control d) Risk transfer Risk of premature death is a a) b) c) d) Financial risk Dynamic risk Personal risk Subjective risk

8.

9.

10.

11. Adverse selection refers to a. Selection of adversities b. Selection of losses c. Selection of risks d. Selection of high risk at low cost 12. Risk Selection in life insurance refers to a. Selection of healthy lives b. Selection of unhealthy lives c. Selection of unhygienic lives d. Selection of hygienic lives

13. Consequences of adverse selection a. Escalation of premiums and complete collapse of an insurance market with the with- drawal of individuals. b. Fall in premiums and Partial collapse of an insurance market

c. Fall in premiums and more number of individuals purchase insurance d. Fall in premiums and complete collapse of an insurance market 14. Insurance covers a. Speculative risks b. Investment risks c. Impure risks d. Pure risks 15. Risk of premature death is a. Personal risk b. liability risk c. property risk d. speculative risk 16. Damage to the property due to floods, a. Personal risk b. liability risk c. property risk d. speculative risk 17. Selling defective medicines create a. Personal risk b. liability risk c. property risk d. speculative risk SECTION A ANSWERS 1.a 2.c 3.c 4.a 5.b 6.d 7.d 8.a 9.d 10. c

11. d 12. a 13. a 14. d 15. a 16. c 17. b SECTION - B 1. What are the characteristics of risk? Ans: The following are the characteristics of risk Risk is the likelihood of an unfortunate occurrence Risk is unpredictable Risk is uncertainty about the future Risk is possibility of adverse deviation from expected outcomes Risk is an outcome that is not favorable All the above definitions of risk highlight one element, namely the presence of uncertainty with regard to the outcome. Uncertainty is subjective as it depends on the individuals perception of risks. It is obvious that no two persons exposed to the same risk will perceive it in the same way. The level of uncertainty will also depend on the information available on the risk. 2. what do you mean by Chance of loss Ans: Chance of loss means the likelihood of the occurrence of an event causing a loss. It is the relative frequency of occurrence of an event resulting in loss. It is the ratio of the number of expected losses to total number of actual losses that actually occurs. Chances of loss = No. Of likely losses / Total no. of possible losses For example, there are 2000 houses in a community. Out of past experience and records, there is a possibility that 20 of these houses will be damaged by fire during a given period of time. Then, the chance of loss due to fire is 1% (20/2000). 3. Describe Physical hazard Ans: Physical hazards are the conditions in which the physical characteristics of an object or an individual tend to increase the frequency of loss.

For example bad weather conditions increase the chances of accidents for aeroplanes, defective materials used in construction of buildings, result in collapse of building and unsafe conditions in the workspace increase the chance of accidents for workers. 4. What is Moral hazard? Give an example. Ans: It is the situation wherein frequency and severity of loss increases due to individuals dishonesty. Moral hazard also occurs due to defective attitude of employees and individuals. Moral hazard can be controlled or reduced by taking appropriate steps and precautions. Dishonest individuals contribute to moral hazards. Examples of moral hazard are giving false information in order to collect the claim, inflating the amount of claim intentionally, faking an accident etc. 5. What is Morale hazard? Give an example. Ans: Moral hazard occurs due to defect in individuals character but morale hazard is due to an individuals indifference or negligence. An individuals indifference to the risk because of the existence of insurance cover constitutes morale hazard. Examples of morale hazards are rash driving, leaving the car unlocked and not checking whether the house doors are locked properly before leaving on a holiday etc. 6. Classify different types of risks Ans: There are different types of risks, which may be classified as follows: Pure and speculative risks. Subjective and objective risks. Fundamental and particular risks Static and dynamic risks 7. Distinguish between Pure risks and speculative risks Ans: Pure risks involve situations where there is a chance of loss or even a breakeven situation but no profit. Pure risks do not have favorable outcomes; at best we are left in the same situation in which we were before the event causing loss occurred. Fires in a go down or factory, an injury at the work place or on the road or a theft or burglary at home are all instances of pure risks. On the other hand, speculative risks may, in addition to the outcomes of a pure risk cited above, have happy or favorable outcomes namely profit or gain. In other words, speculative risks can have three possible outcomes viz. loss, breakeven situation or gain. A commonly cited example of speculative risk is investment in shares in which all the three outcomes are possible.

The difference between pure and speculative risks is highlighted to make the point that pure risks are generally insurable but not speculative risks, as the law of large numbers is applicable to pure risks and not to speculative risks. 8. Distinguish between subjective risks and objective risks Ans: Subjective risk arises out of an individuals mental state. It is the result of individuals uncertainty about the outcome of an event. Subjective risks occur due to psychological fear about the outcome of an event whether it will be favorable or not to an individual. Objective risks as the term suggests, can be observed and for that reason capable of measurement unlike subjective risks which are not measurable. Here the probability of an event occurring can be determined in two ways: One by deductive reasoning (as in the case of a balanced coin being tossed) or by inductive reasoning (as in the case of an actuary determining the probability of death of a person at a certain age). 9. Distinguish between Fundamental risks and particular risks Ans: Fundamental risks are group risks, which occur due to social, economic and political changes in the country. Large numbers of people are affected by these fundamental risks. Fundamental risks also arise due to natural calamities. Examples: Crisis in the economy, widespread unemployment, sudden wars, poverty, dramatic changes in government policies, inflation, floods, earthquakes, famine, volcanic eruptions and other natural calamities are also described as fundamental risks. Particular risks on the contrary are personal in nature and affect only individuals. These arise from individual causes and the consequences affect individuals. Examples: Injury due to accidents, poor health condition, robbery, damage due to fire etc., Generally governments formulate certain programmes to deal with fundamental risks whereas affected individuals deal with particular risks by using suitable risk management techniques.

10. Distinguish between Static risk and dynamic risk Ans: Static risks can be pure or speculative. Natural events such as storm snowfall and death are described as pure static risks. Industries in a stable economy are a good example of static speculative risk.

Dynamic risks can also be pure or speculative risks. Dynamic risks will affect the whole economy. These risks are beyond the control of an individual. Dynamic risks arise out of socio economic changes such as globalization of economy, impact of IT, information explosion and other similar major changes affecting society. Due to dynamic risks, society often benefits but only in the long term. 11. What are the sources of pure risk? Ans: The sources of pure risks include the following: Personal risks Property risks Liability risks Risks arising out of failure of others 12. Describe the methods of Handling Risk Ans: The different methods of handling risk are. Risk avoidance Controlling losses Risk retention Transfer of risk 13. what are the steps in the risk management process? Ans: The steps in the risk management process are Setting objectives Risk identification Evaluation of risks Selection of appropriate risk management technique Implementation of risk management technique Review the risk management decision 14. Distinguish between Risk Management and Insurance Management Ans: Risk management as it is being practiced today is of relatively recent origin. Even till the middle of the 20tcentury, it was customary to call the risk manger as the insurance manager and his department was referred to as insurance department. His activities were mainly focussed on insurance buying. Today most large companies have a full time senior executive designated as Risk Manager. His job is clearly defined. He is a specialist whose job is to identify and analyse the risks his company is exposed to, select methods with which these risks may be controlled or eliminated and advise top management on insurance buying to indemnify the company against insurable losses to reduce the impact of potential financial loss. Thus even though insurance buying may

remain a major area of his work, it is not the only area of operation. Risk management is a much wider concept than insurance management. Insurance Management Vs Risk Management 1. Believes exclusively in insurance as 1. Makes a systematic study of all risks the risk management tool. an organization is exposed to, evaluates them and identifies the best and most economical method of handling each risk. 2. Expertise is in the area of insurance 2. Has a better understanding of risk in and tends to look at risk solely from an the company as a professional in risk insurance angle management 3. Covering all or most of the risk 3. Looks at risk management from the through insurance would be a heavy cost angle and considers various cost for the company options including insurance for the management of risks. Hence cost effective 4. Insurance management is of limited 4. Handles risks for the company by use to an enterprise in managing risks. adopting integrated risk management or enterprise risk management practices SECTION C CASE STUDIES 1. Vamsi is a college student. He owns a high-mileage bike that has a current market value of Rs. 80,000. The current replacement value of his clothes, television set, stereo set, and other personal property in a rented apartment total Rs. 1 lakh. He wears disposable contact lenses, which cost Rs. 6,000 for a six-month supply. He also has a waterbed in his rented apartment that has leaked water in the past. An avid runner, Vamsi runs five miles daily in nearby public park that has the reputation of being extremely dangerous because of drug dealers, numerous assaults and muggings, and drive-by shootings. Vamsi parents both work to help him pay his tuition. For each of the following risks or loss exposures, identify an appropriate risk management technique that could be used to deal with the exposure. Explain your answer.

a. Physical damage to the Bike because of a collision with another motorist b. Liability lawsuit against Vamsi arising out of the negligent operation of his bike. c. Total loss of clothes, television, stereo, and personal property because of a grease fire in the kitchen of his rented apartment d. Disappearance of one contact lens e. Waterbed leak that causes property damage to the apartment f. Physical assault on Vamsi by gang members who are dealing drugs in the park where he runs g. Loss of tuition from Vamsis father who is killed by a drunk driver in an auto accident. Ans: a. Transferring Risk to insurance company by taking motor insurance b. Transferring Risk to insurance company by taking third party motor liability insurance c. Transferring Risk to insurance company by taking Household property insurance. d. Either avoid the risk of losing the lens or retaining the risk with him e. Avoidance of using water bed because it has leaked water in the past or get it repair that water bed to control the losses. f. Avoidance of running in the park because it is notorious of drug dealings. g. Transferring risk by taking childrens education policy by parents. 2. Identify the types of financial losses likely to be incurred by each of the following parties. a. A person who negligently injures another motorist in an auto accident b. A restaurant that is shut down for six months because of a tornado c. A family whose family head dies prematurely d. An attorney who fails to file a legal brief on time for a client e. A tenant whose apartment burns in a fire Ans: a) Liability loss b) Property loss / consequential loss c) Personal loss d) Liability loss e) No financial loss to tenant with regards to apartment but personal loss to his belongings.

CHAPTER 2 INTRODUCTION TO INSURANCE

SECTION A
1. The economic principle of life insurance is on the basis of a. the law of large numbers b. the theory of probability c. the pooling of resources d. utmost good faith e. subrogation 2. Life insurance contract provides a. economic security b. emotional security c. legal security d. physical security e. environmental security 3. The most essential feature of a life insurance contract is a. sharing of the losses b. replacing uncertainty through certainty c. to bring the insured to the same position that existed prior to the event of losses d. Both (a) and (b) above e. All of (a), (b), and (c) above 4. The economic theory of insurance is based on the a. concept of spreading risk b. theories of probability c. law of large numbers d. Both (a) and (b) above e. Both (a) and (c) above 5. The amount of human life value is calculated by properly discounting the a. current earnings of the insured b. fraction of the income of the insured that is used for the dependents c. saving potential of the insured d. potential of consumption expenditure of the insured e. social status of the insured

6. For the calculation of human life value, if the rate of discount increases, the economic value will a. decrease b. increase c. remain the same d. becomes zero e. be affected but it is impossible to predict. 7. Needs approach does not consider the a. clean-up funds b. readjustment income c. income to support the family d. improvement of social status e. retirement needs 8. Which of the following is incorrect with reference to life insurance policy? a. It is a compulsory savings option to the policy holder. b. It assures the repayment of the borrowings against the unfortunate death of the policyholder c. It attracts suitable stamp duty for the transfer of property in a life insurance d. It can be used a collateral in case of some types of property dealings e. It allows income tax rebate to the policyholder. 9. Which of the following can be an advantage for taking a life insurance policy? a. It provides security in case of an unexpected contingency. b. It provides the safety net for the repayment of a loan taken for the dream home c. It assures the creditors for the repayment of the principal and interest of any loan taken by the policyholder. d. Both (b) and (c) above e. All of (a), (b) and (c) above 10. Advantage of taking a life insurance policy is that a. it cannot help to develop an estate as chosen by the policyholder b. it does not encourage to withdraw the savings prematurely c. it is of no use to acquire a dream home d. income tax benefit is not allowed for taking the life insurance coverage e. None of the above

11. When are the proceeds of a term insurance policy payable a. only on death during the term b. Only on survival till the end of the term c. Anytime during the term of the policy d. Never payable e. None of the above 12. Under which policy is the sum assured payable in case of survival only till the end of the term? a. Term insurance policy b. Term endowment policy. c. Pure insurance policy. d. Pure endowment policy. e. Endowment insurance policy. 13. What is the nature of the term insurance policies? a. Low cost - high risk. b. High cost low risk. c. Moderate cost moderate risk. d. The best choice for a savings plan. e. No use in the life risk coverage. 14. Till when under the whole life insurance policy is the policy coverage available? a. Up to certain age of the policyholder b. For a tenure of twenty years c. Like that of a pure endowment policy. d. Till the death of the insured. e. Till the retirement of the insured.

SECTION A ANSWERS

1. (c ) As per the economic principle of life insurance, within a substantially large population, all the members are facing the same kind of risk. All are likely to suffer the losses equally but only a few of them will have to face the harsh reality. But who are those unfortunate persons, that cannot be predicted in advance through human intelligence. Hence, the concept of insurance operates by pooling resources. 2. ( a) Life insurance contract provides the necessary economic support in case of the occurrence of premature death or excessive longevity of an individual, by which one can get the security of cash flows for self as well as for the dependents in case of disability or excessive longevity; and for the dependants in case of premature death. 3. (d) In any insurance contract, the losses of the unfortunate few are shared by a large number of persons facing similar types of risks. Therefore, the financial uncertainties of those unfortunate few are shared by a large number of individuals in order to assure certainty of cash compensation from the insurer. But a life insurance company cannot restore the insured to the same position prior to the event of losses. 4. (e ) In any insurance business, the small amount of premium is charged upfront from the insureds who may suffer similar type of losses. But finally, only the unfortunate few insureds will face the harsh reality that is to be indemnified from those premiums. Therefore, the risks are spread. But, the application of the law of large numbers finally leads to the accuracy of estimation of losses. Theories of probability are used to estimate the amount of premium under actuarial principle. 5. (b) The concept of human life value is used to evaluate the requirement of life insurance coverage of an individual. The objective of such estimation is to ensure the same standard of living to the dependents of a person even after his pre-mature death. 6. (a) As the discount rate increases, the denominators of each factor will go up, while the numerators will remain the same. Therefore, the ultimate human life value will decrease.

7. (d) Clean up funds are required to pay off the debts and the readjustment funds are used to maintain the same standard of living. Income to

support the family is meant to support the necessities of the children and wife. Lastly, retirement needs consider the financial requirement at the old age. But the need for improvement of social status is not considered. 8. ( c) Under Section 38 of the Insurance Act, 1938, transfer of property in a life insurance policy can be done simply by an endorsement on the back of the policy document. 9. (e) Life insurance policy coverage can be taken in all such cases as mentioned above. 10. (b) Life insurance is a type of forced and compulsory savings to the policyholder. In other types of savings, one may withdraw the accumulated sum at the time of even a minor liquidity crisis. But in case of life insurance, the policyholder has to suffer a substantial amount of financial loss to sacrifice the insurance coverage. 11. (a) The very nature of the term insurance policies is such that the beneficiary proceeds are payable only on death during the selected term and nothing is payable in case of survival of the insured until the end of the contracted term. 12. (d) In case of pure endowment policy, the policyholder receives the policy value only on survival till the end of the policy term. Otherwise, nothing is receivable from the insurer. 13. (a) Term insurance policies only consider the risk premium, operational expenses and a profit factor. Therefore, due to the absence of any savings component, they are relatively cheaper. 14. (d) Whole life insurance policy is the longest term insurance policy. In this case, the proceeds of the policy are remitted by the insurer following the death of the insured, irrespective of its timing.

SECTION - B
1. Define Insurance. Ans. Insurance can be defined both from an economic perspective and from a legal perspective. From an ECONOMIC PERSPECTIVE, insurance is a financial intermediation function by which individuals exposed to a specific financial contingency each contribute to a pool from which covered events suffered by participating individuals are paid. From a LEGAL PERSPECTIVE , insurance is an agreement by which one party, the policy owner, pays a stipulated consideration called the ' premium' to the other party called the insurer in return for which the insurer agrees to pay a defined amount of money or provide a defined service if a covered event occurs during the currency of the policy. 2. What are the basic characteristics of insurance? Ans : The basic characteristics of insurance are Pooling of losses The law of large numbers Payment of fortuitous losses Risk transfer Indemnification 1.Pooling of losses--Pooling of risks or the sharing of losses is the heart of insurance .Pooling is spreading of losses incurred by the few over the entire group, so that, in the process, average loss is substituted for actual loss. It implies 1. The sharing of losses by the entire group, and 2 prediction of future losses with some accuracy based on the law of large numbers. 2.The law of large numbers-- Many lines of insurance risk reduction are based on the law of large numbers. It states that the greater the number of exposures, the more closely will the actual results approach the probable results that are expected from an infinite number of exposures. 3.Payment of fortuitous losses Payment is made for losses that are unforeseen and unexpected and occurs as a result of chance. Means the loss must be accidental.

4. Risk transfer-- In this, a pure risk is transferred from the insured to the insurer who typically is in a stronger financial position and is willing to pay the

loss than the insured {ex. The risk of premature death, poor health, destruction or theft of property} 5. Indemnification--means the insured is restored to his or her approximate financial position prior to the occurrence of the loss 3. Write a brief note on the Law of Large Numbers Ans: Many lines of insurance risk reduction are based on the law of large numbers. The law states that the greater the number of exposures, the more closely will the actual results approach the probable results that are expected from an infinite number of exposures. If there are a large number of exposure units, the actual loss experience of the past may be a good approximation of future losses. To charge premium that will be adequate for paying all losses , expenses and margin for profit, this concept is important to insurer because he can predict future losses with a greater degree of accuracy as the number of exposures increases. 4. What do you mean by Risk Transfer? Ans. In this a pure risk is transferred from the insured to the insurer, who typically is in a stronger financial position and is willing to pay for the loss than the insured {ex. The risk of premature death, poor health, destruction or theft of property}. 5. What do you mean by Indemnification? Ans. Indemnification means the insured is restored to his or her approximate financial position prior to the occurrence of the loss. 6. What are the requirements of an insurable risk? Ans. Insurers normally insure only pure risks. However all pure risks are not insurable. From the viewpoint of the insurer the requirements of an insurable risk are There must be a large number of exposure units. The loss must be accidental and unintentional The loss must be determinable and measurable The loss should not be catastrophic. The chance of loss must be calculable The premium must be economically feasible 7. How does insurance differ from gambling?

Ans. Insurance is often confused with gambling. But there are two important differences between them. 1. Gambling creates a new speculative risk, while insurance is a technique for handling an already existing pure risk. 2. Gambling is socially unproductive, because the winner' s gain comes at the expense of the loser. In contrast, insurance is always socially productive, because neither the insurer nor the insured is placed in a position where the gain of the winner comes at the expense of the loser. The insurer and the insured both have a common interest in the prevention of a loss. Both parties win if the loss does not occur. 3. Gambling transactions never restore the losers to their former financial position. In contrast insurance contracts restore the position of insureds financially in whole or in part if a loss occurs. 8. How does Insurance differ from Speculation? Ans. Insurance is different from speculation. Insurers insure pure risks. Pure risk is defined as a situation in which there are only the possibilities of loss or no loss. While speculation deals with speculative risk. Speculative risk is defined as a situation in which either profit or loss is possible. 2. The law of large numbers which is the basic characteristic of insurance can be applied more easily to pure risks than to speculative risks. Based on this, insurers predict future loss experience. 3. Finally, society may benefit from a speculative risk even though a loss occurs, but it is harmed if a pure risk is present and a loss occurs. 9. How does insurance differ from Hedging? Ans. Though both insurance and hedging techniques are similar in that risk is transferred by a contract, and no new risk is created, there are major differences between them. First, an insurance transaction involves the transfer of insurable risks, because the requirements of an insurable risk can generally be met. However, hedging is a technique of handling risks that are typically uninsurable, such as protection against decline in the price of agricultural products and raw materials.

Second, insurance can reduce the objective risk of an insurer by application of law of large numbers. In contrast, hedging typically involves only risk transfer, not risk reduction. The risk of adverse price fluctuations is transferred to speculators who believe they can make a profit because of superior knowledge of market conditions. The risk is transferred, not reduced, and prediction of loss generally is not based on the law of large numbers. 10. What are the types of Insurance ? Ans. TYPES OF INSURANCE SOCIAL INSURANCE {Emphasis on social equity through income redistribution} * OASDI * MEDICARE * UNEMPT. LIFE Whole life [death] PRIVATE INSURANCE focuses on individual equity

HEALTH Disability income incapacity] Medical expense[health, accident, injury.

NON -LIFE

Long term care

Workers compensation

11. Differentiate between Life and Non-Life insurance Ans. LIFE INSURANCE is a branch of private insurance and includes the following four classes of insurance coverage: 1. death: called life insurance \ assurance 2. living a certain length of time : called endowments, pensions and annuities. 3. incapacity: called disability and long term care 4. injury or incurring a disease: called health insurance, accident insurance, and medical expense insurance.

Property loss

Liability loss

Endowment

Annuity

Term

NON-LIFE INSURANCE is also a branch of private insurance and includes the following coverage. 1. property losses: damage or destruction of homes 2. liability losses: payments due to professional negligence 3. workers compensation and health insurance in some countries 12. What are the different uses of Life and health insurance to individuals? Ans. The prime use of life and health insurance for individuals, families and businesses is to provide financial protection. The other uses are Assists in making saving possible Furnishes a safe and profitable investment Encourages thrift Minimizes worry and increases initiative Furnishes an assured income in the form of annuities Helps preserve an estate

13. How does society benefit from the insurance? Ans. The major social and economic Benefits of insurance include the following. Indemnification for loss: Indemnification permits individuals, families and business firms to be restored to their former financial position after a loss occurs. The community also benefits because its tax base is not eroded. Less worry and fear: Worry and fear are reduced for a family and property owner both before and after a loss because the insureds know that they have insurance that will pay for the loss. Source of investment funds: The insurance industry is an important source of funds for capital investment and accumulation. Investments increase society' s stock of capital goods, and promote economic growth and full employment. Insurers also invest in social investments, such as housing, nursing homes and economic development projects. In addition, the total supply of loanable funds is increased by the advance payment of insurance premiums, the cost of capital to business firms that borrow is lower than it would be in the absence of insurance Loss prevention: Society benefits with the involvement of insurance companies directly and indirectly in the loss prevention activities such as

Highway safety and reduction of automobile deaths. Fire prevention. Reduction of work related disabilities. Prevention and detection of arson losses. Prevention and defective products that could injure the user. Prevention of auto thefts. Prevention of boiler explosions. Educational programs on loss prevention. Enhancement of credit: Insurance enhances a person' s credit. And it makes a borrower a better credit risk because it guarantees the value of the borrower' s collateral or gives greater assurance that the loan will be repaid. 14. HOW MIGHT ONE ASSESS THE IMPACT OF THE LIFE INSURANCE INDUSTRY ON THE GENERAL ECONOMY? Ans. Conference on Trade and Development [UNCTAD] noted that an efficient insurance market can aid in overall economic development in the following ways. It can contribute to social stability by permitting individuals to minimize financial stress and worry. It can reduce the financial burden on the state of caring for the aged and destitutes. It can benefit the economy by creating a source of financing for new businesses, housing and farming etc. It generates employment. It can permit more favorable terms to borrowers, can decrease the risk of default and can minimize financial disruption due to death of key employee and owner. It can promote better employee / employer relations and can provide low cost benefits to a broad spectrum of persons who may otherwise have been unable to obtain such protection. INSURANCE AND ECONOMIES: A study by the Organization for Economic Cooperation and Development[OECD] observes that Insurance provides at least three categories of services important to economies. Those are Substitute for Government security programs -- The study of OECD countries found a significant negative relationship between social expenditures and life insurance premiums.

Mobilizes savings -- Role of savings in economic development is crucial. Insurers act as financial intermediaries and reduce transaction costs between savers and borrowers, create liquidity and facilitate economies of scale. Fosters a more efficient capital allocation Insurers gather substantial information to provide funds to the most attractive firms, and projects, show continuing interest on them, monitor entrepreneurs and foster more efficient allocation of a country' s scarce financial capital. 15. Insurance entails certain costs to Society Explain Ans: THE COSTS OF INSURANCE: Although insurance offers societies great social and economic benefits, it also carries certain costs. These are Costs of doing business. Fraudulent claims. Inflated claims. Costs of doing business: One important cost is the cost of doing business. An expense loading [the amount needed to pay all expenses, including commissions, general administrative expenses and an allowance for contingencies and profit.] must be added to the pure premium to cover the expenses incurred by companies in their daily operations. However, these additional costs can be justified from the insured' s view-point of engaging in a wide variety of loss prevention activities. And from the industry view-point, it provides jobs to millions of workers. Fraudulent Claims: A second cost of insurance encourages moral hazard like Faked auto accidents. Fake slip and fall accidents . Phony burglaries, thefts, vandalism. False health insurance claims. Murders etc. These social costs fall directly on society. Inflated Claims: Submission of inflated or ' padded' claims though loss is not intentionally caused by the insured like Attorneys for Plaintiffs sue for high liability judgements than the true economic loss of the victim. Inflate the amount of damage in automobile collision claims. Disabled persons often manage to collect disability income benefits for a larger duration.

In summary the social and economic benefits of insurance generally outweigh the social costs. Insurance reduces worry and fear, it contributes its service to economic and social stability and provides financial security to individuals and firms. So the social costs of insurance can be viewed as the sacrifice that society must make to obtain these benefits. 16. Explain the economic bases of life and health insurance Ans. The economic bases of life and health insurance arise out of family purposes or from business purposes .We can classify the FAMILY PURPOSES as sources of the economic value of the Human life preservation of family economic security. moral obligation to provide protection. BUSINESS PURPOSES Key person indemnification credit enhancement business continuation employee benefit plans NEEDS APROACH: Under the concepts which influence demand for insurance this needs approach is having practical value. Mcgill segmented this needs into Clean up Fund [hospital, doctors, nurses bills, burial expenses, legal fees etc.] Readjustment shock [ to cushion the economic and emotional shock] Critical period income for children. Life income for surviving dependent spouse. Special needs [ mortgage redemption, educational needs, emergency needs[illness, surgery etc.] Retirement needs Hence the economic bases of insurance determine how much life and health insurance should be carried like any other product.

17. EXPLAIN 'HUMAN LIFE VALUE CONCEPT' AND IT'S SIGNIFICANCE TO LIFE INSURANCECE. Ans. Human life value: The human life value concept is one segment of the general theory of human capital. The relationship between life insurance and human capital has been acknowledged through the human life value concept. In 1924, the late Dr.S.S.Huebner proposed this concept. It measures the actual future earnings or value of services of an individual i.e. the capitalized value of an individual' s future net earnings after subtracting self maintenance cost such as food, clothing and shelter. In connection with life and health insurance, the economic value of a human life is derived from both a.}its earning capacity and b.}the financial dependence of other lives {or Organizations} on that earning capacity

SECTION C CASE STUDIES


1. There are numerous definitions of insurance. Based on the definition of insurance stated in the text, indicate whether each of the following guarantees is considered insurance. a. A television set is guaranteed by the manufacturer against defects for 90 days. b. A new set of radial tires is guaranteed by the manufacturer against road defects for 50,000 miles. c. A builder of new homes gives a ten-year guarantee against structural defects in the home. d. A consigner of a note agrees to pay the loan balance if the original debtor defaults on the payments. e. A large group of homeowners agree to pay for losses to homes that burn during the year because of fire. 2. The existence of insurance prompts some insureds to deliberately cause a loss so as to profit from insurance. These social costs fall directly on society. Give some examples of such fraud. 3. Does the Risk of Fire fulfill the requirement of an insurable risk? 4. Does the Risk of unemployment fulfills the requirement of an insurable risk.

SECTION C ANSWERS 1. (a) - No (b) No ( c) No (d) No (e) Yes 2. a) Creating a fraudulent claim Staged or fake auto accident. Staged slip-and-fall accident. False claim of foreign object in food or drink. Faking a death to collect benefits. Murder for profit. Phony burglary, theft, or vandalism. Arson or intentional water damage. Staged theft of auto. Staged homeowner accident or burglary. b) Overstating amount of loss Inflating bodily injuries in auto accident. Inflating value of terms taken in burglary or theft. Inflating damage claim from minor fender bender. Medical providers inflating billing or coding of medical procedures. c) Misrepresenting facts to receive payment. Claiming preexisting damage occurred in current accident. Claiming damages to auto when none occurred. Claiming minor injury creates partial or total disability. Receiving disability payments and working elsewhere. d) Misrepresentation to receive payments. Claiming false disability. Providing unnecessary medical treatment. Charging for medical tests not carried out. Upcoding for medicine by issuing generic pills and charging for name brands Personal injury mills of doctors, lawyers, and claimants.

e) Misrepresentation to obtain a policy or lower premiums. Misrepresenting health information on life insurance and then submitting a false claim.

Misrepresenting name, date of birth, or Social Security number and then submitting a false claim. f) Insider and internal fraud Agent or insurer pocketing premiums, then issuing no policy or a bogus policy. Agent or insurer issuing fake policies, certificates, ID cards, or binders. Agent or insurer making false entry on document or statement.

3. Yes.

Requirements Does the risk of fire satisfy the requirements? 1. Large number of Yes. Numerous exposure units are present exposure units 2. Accidental and Yes. With the exception of arson, most fire losses are accidental and unintentional unintentional loss 3. Determinable and Yes. If there is disagreement over the amount paid, a property insurance policy has provisions for measurable loss resolving disputes. 4. No loss catastrophic Yes. Although catastrophic fires have occurred, all exposure units normally do not burn at the same time.

5. Calculable chance Yes. Chance of fire can be calculated, and the average severity of a fire loss can be estimated in of loss advance. 6. Economically Yes. Premium rate per $100 of fire insurance is feasible premium relatively low.

4. Yes. Requirements 1. Large number of exposure units Does the risk of unemployment satisfy the requirements? Not completely. Although there are a large number of employees, predicting unemployment is difficult because of the different types of unemployment and labor. No. A large proportion of unemployment is due to individuals who voluntarily quit their job. Not completely. The level of unemployment can be determined, but the measurement of loss is difficult. Some unemployment is involuntary; however, some unemployment is voluntary. No. A severe national recession or depressed local business conditions could result in a catastrophic loss. No. The different types of unemployment generally are too irregular to estimate the chance of loss accurately. No. Adverse selection, moral hazard, and the potential for a catastrophic loss could make the premium unattractive.

2. Accidental and Unintentional loss 3. Determinable and measurable loss

4. No catastrophic loss 5. Calculable chance of loss 6. Economically feasible premium

CHAPTER 3 THE FUNDAMENTAL LEGAL PRINCIPLES OF GENERAL AND LIFE INSURANCE


SECTION - A
1. How is the legality of the object of a contract satisfied in case of a life insurance contract? a. b. c. d. e. Through the application of the principle of indemnity. Through the application of the principle of subrogation. Through the application of the principle of insurable interest Through the application of the principle of utmost good faith. Through the application of the principle of proximate cause.

2. What type of fact may be considered as material fact in case of an insurance contract? a. b. Facts that tend to render a risk proposed greater than normal. Facts necessary to explain the exceptional nature of risk proposed for insurance where, without them, the insurer will justifiably believe that the risk is normal. Facts which appear to suggest some special motive for insurance. Facts which show that the proposer himself is in some way abnormal All the above.

c. d. e.

3. Which of the following facts is/are not required to be disclosed by the proponent? a. b. c. d. e. Facts that are a part of law Facts that reduce the risk of losses. Facts that are published in the newspapers. Facts which can be obtained from the information provided by the insured. All of the above

4. What is the difference between representation and warranty? a. A representation need only be substantially correct while a warranty must strictly and literally be complied with b. An insurer cannot repudiate its liability against an improper representation, unless that is proved to be a material one while any thing wrong in the warranty is sufficient to avoid the liability. c. A representation does not lead to any risk to the policyholder while a warranty may lead to serious contingency to the policyholder. d. Both (a) and (b) above e. All of (a), (b) and (c) above 5. In which case the principle of indemnity is applicable? a. b. c. d. e. All types of insurances Non-life insurance only. Life insurance only Either (b) or (c) above None of the above

6. In case of life insurance contract, how is the concept of indemnity followed? a. b. c. d. e. Through the assessment of premium paying capacity. Through the assessment of insurance coverage requirement. Through the assessment of the tenure of insurance. Both (a) and (b) above All of (a), (b) and (c) above

7. What is the similarity between insurance and wagering contract? a. b. c. d. e. Both are legally enforceable. Both types of contracts need the existence of insurable interest. Both types of contracts assess the possibility of losses. Both types of contracts follow the principle of utmost good faith. In both types of contract, the counter parties will suffer the same type of losses.

8. X is presently suffering from tuberculosis but in the proposal for obtaining the required insurance coverage at the normal rate, he did not mention it. It is an act of a. b. c. d. e. Coercion Undue influence Fraud Misrepresentation Mistake

9. Which of the following relationships does NOT have any insurable interest? a. b. c. d. e. Proponent and his own life Husband and wife. Father and son. Debtor and creditor Employer and employee.

10. Which of the following relationships leads to limited amount of insurable interest? a. b. c. d. e. Insurer and Insured. Partners in a business. Debtor and creditor. Both (b) and (c) above. All of (a), (b), and (c) above.

11. In a typical case, the proponent of a life insurance contract wrote the age wrongly while the insurer issued a policy other than the applied one. What will be the impact of thee in the contract between the insurer and insured? a. It is a case involving coercion and hence the contract will be null and void. b. It involves undue influence of the insurer on the insured and hence no contract is admissible between the parties. c. It is a matter of misrepresentation by the insurer and so voidable at the wishes of the insurer. d. The proponent is involved in the act of fraud and so the insurer can sue him. e. Both the parties have committed mistakes and hence the contract is unenforceable.

SECTION A ANSWERS
1. (c ) Principle of insurable interest is the most important feature of a life insurance contract to maintain the legality of the object of the contract. If the existence of the insurable interest is not considered properly, it will amount to gambling. 2. (e) All the given situations will place the insurer in a more risky position which is a essential part for making the underwriting decisions by the insurer. 3. (e) These facts are expected to be known to the insurers owing to their level of knowledge and understanding as well as their underwriting skills. Hence, the proponent need not be worried about their disclosures. 4. (d) Any representation made by the policyholder can lead to the risk of misrepresentation while a warranty may lead to a serious contingency to the policyholder. 5. (b) The principle of indemnity means making good the losses suffered by an unfortunate person. But in a life insurance contract, it is impossible to assess the value of life of an individual and so the principle of indemnity cannot be applied. 6. (d) One cannot buy a life insurance policy for any unlimited amount at any point of time. The insurance company assesses the premium paying capacity and the insurance coverage requirement of the proponent. This is done in order to ensure that the insurance coverage should not be cumbersome to the insured. Simultaneously, the insurance proceeds should be sufficient enough to maintain the same standard of living, following the stoppage of income of the insured. 7. ( c) In a life insurance contract, the insurer assesses the possibility of the losses from the mortality tables. In a wagering contract, the possibility of losses is estimated either by empirical judgement or analytically depending on the situation.

8. ( c) Fraud occurs when the proposer intentionally suppresses the material information at the time of submission of an offer. In the life insurance business, the disease of tuberculosis seriously affects the risk profile of the insured and hence it may be considered as material information to the insured. 9. ( c) Except the proponent and his own life, as well as husband and wife, no other emotional relationship leads to any insurable interest. 10. (e) The relationships in all the given cases lead to limited amount of insurable interest. In the first case, the insurer is liable to provide the contracted benefits on death or disability of the insured as contracted. While in the other two cases, the extent of insurable interest is limited to the amount of capital brought by the partner to the business and the amount outstanding in the books of the creditor. 11. (e) If both the parties to a contract committed mistakes at the time of making the contract, the contract will be treated as null and void. As a result of that, neither of the party will be liable to other regarding the execution of the contract since the essential ingredient of a contract i.e. consensus ad idem is missing in this case due to the acts of both the parties.

SECTION - B
1. What are the fundamental principles related to General and Life Insurance Contracts? Ans. The most important fundamental principles related to General and Life Insurance Contracts are 1) 2) 3) 4) 5) 6) utmost good faith indemnity insurable interest subrogation contribution proximate cause

2. Explain why an insurance contract is considered of utmost good faith and not of caveat emptor? Ans. An insurance contract is based on the principle of utmost good faith. The principle of utmost good faith is supported by three important legal doctrines: representations, concealment [intentional failure of the applicant for insurance to reveal a material fact to the insurer] and breach of warranty [promise made by the insured in the contract]. Each party [ insured and insurer] to the contract is entitled to rely on good faith upon the representations of the other. The rule of caveat emptor [let the buyer beware] does not generally apply. The Insurer believes the representations of the Insured. Representations of insured are statements of his or her age, weight, height occupation, state of health, family and personal history. And from insurer' s side - it is intricate and highly technical. Here both the parties are under an obligation not to attempt to deceive or withhold material information from the other. The insurance contract is voidable at the insurer' s option if the representation is material, [if the insurer knows the true facts, the policy would not have been issued or would have been issued on different terms] false, reliance[ the insurer relies on the misrepresentation in issuing the policy at a specified premium] and innocent or unintentional misrepresentation. 3. Explain the principle of indemnity. And what are the exceptions to this principle of indemnity? Ans. It is one of the important legal principle in Insurance. The principle of Indemnity states that the insurer agrees to pay no more than the actual amount of the loss, which means the insured should not profit from a loss. And this principle has two fundamental purposes.1. To prevent the insured from profiting from a loss and 2.to reduce moral hazard. This indemnity is different for different types of insurance. In property insurance , the basic method for indemnifying the insured is based on the actual cash value of the damaged property at the time of loss and this cash value can be determined through 3 major methods such as 1.replacement cost less depreciation 2 fair market value 3 broad evidence rule. In liability insurance, the insurer pays upto the policy limit. In life insurance, the amount paid when the insured dies is the face value of the policy Life insurance is not a contract of indemnity.

In business income insurance, the amount is paid on the loss of profits and continuing expenses when the business is shut down because of loss from a covered peril. Exceptions: The major exceptions to the principle of indemnity are 1.valued policy [pays the face amount ex. Payment for the loss of antiques.] 2.valued policy laws [ differ from state to state] 3. Replacement cost insurance [ no deduction for depreciation ] 4. Life insurance [ a valued policy ] 4. What problems might arise if life policies were contracts of indemnity and property and liability policies were valued contracts? Ans. The principle of Indemnity states that the insurer agrees to pay no more than the actual amount of the loss. In the case of life policies the amount is paid when the insured dies. The actual amount of loss of human life value is capitalized through the face value of its policy. It is only a valued contract and it cannot be a contract of indemnity. If it is a contract of indemnity, the principle of paying actual amount of the loss can not be operated. 5. What is an insurable interest? Why is an insurable interest required in every insurance contract? Ans. It is an important legal principle. It states that the insured must be in a position to lose financially if a loss occurs, or to incur some other kind of harm if the loss takes place. To prevent gambling, to reduce moral hazard and to measure the amount of the insured's loss [ in property insurance] all insurance contracts must be supported by an insurable interest. There is a difference between an insurable interest in property and liability insurance and life insurance. IN PROPERTY AND LIABILITY INSURANCE a. Ownership of property can support an insurable interest because he loses financially if his property is damaged. b. Potential legal liability also can support an insurable interest in the property of the customer because these firms are legally liable for damage to the customers'goods caused by their negligence. c. Secured creditors also have an insurable interest in the property pledged to them. d. A contractual right also can support an insurable interest. e. In property insurance, the insurable interest must exist at the time of the loss.

IN LIFE INSURANCE there is no question of insurable interest if the life insurance is purchased on their own life. If the life insurance policy is purchased on the life of another person this person should have an insurable interest on that person' s life. Close ties of blood or marriage or a pecuniary interest will satisfy the insurable interest requirement in life insurance. And this requirement must be met only at the inception of the policy and not at the time of death. Life insurance is not a contract of indemnity but is a valued policy that pays a stated sum upon the insured' s death. 6. Explain the principle of subrogation. Why is subrogation used? Ans. The principle of subrogation strongly supports the principle of indemnity. According to George E. Rejda subrogation means substitution of the insurer in place of the insured for the purpose of claiming indemnity from a third person for a loss covered by insurance. Ex. In an accident the insured victim gives legal rights to the insurer to collect damages from the negligent third party instead of collecting himself directly from the third party. The main purposes of subrogation are a. to prevent the insured from collecting twice for the same loss, b. to hold the negligent person responsible for the loss and c. to hold down insurance rates. Insurer must pay before he claims subrogation. The insured cannot impair the insurer' s subrogation rights. The insurer can waive its subrogation rights in the contract either before or after the loss. Subrogation does not apply to life insurance and to most individual health insurance contracts because life insurance is not a contract of indemnity and subrogation relates to only contracts of indemnity. Finally insurer cannot subrogate against its own insured because it defeats the basic purpose of purchasing insurance. 7. What are the differences between subrogation and contribution? Ans. The principle of subrogation strongly supports the principle of indemnity. According to George E. Rejda subrogation means substitution of the insurer in place of the insured for the purpose of claiming indemnity from a third person for a loss covered by insurance. Subrogation and Contribution are corollaries to the principle of indemnity. Both these arise only in property insurance. Contribution arises with the liberty of double insurance which means the assured can insure the same property with more than one insurer and that too with overinsurance. The conditions to satisfy this right of contribution are -- all the insurance must relate to the same subject matter, they should cover the same interest of the same insured, they should cover same peril, and all of them should

be in force at the time of loss. When all these conditions are satisfied then the insurer who has paid first in the full the assured, can claim contribution from the other co insurers. And this claim amount depends on the aggregate of rateable clause or continental law. Though the doctrines of subrogation and contribution are important corollaries of indemnity they differ with each other. Subrogation The loss shifts from one person to another It is against third party One insurer and one policy The right of the insured is claimed Contribution The loss is distributed among the insurers It is in between insurers. More than one insurer The right of the insurer is claimed.

8. What is the legal doctrine of proximate cause? Ans. The legal doctrine of proximate cause is based on the principle of cause and effect. And it does not concern itself with the cause of causes. The law provides the rule " causa proxima non remota spectatur " which means to be proximate, a cause must be immediate cause, which is effectual in producing that result but not the remote or distant one. And this cause has to be selected by applying common sense standards i.e. the standards of a man in the street. An insurance policy is designed to provide compensation only for insured perils [ named in the policy as insured ex. Fire, theft etc.] but not for uninsured [ not mentioned in the policy ] and excepted or excluded [ stated in the policy as excluded ] perils. And the liability of the insurer arises only if the loss is caused by an insured. The selection of proximate cause is not an easy and simple task because loss may be caused by several events acting simultaneously or one after the other. It is necessary to differentiate between the insured peril, the excepted peril and the uninsured peril. Application of the doctrine: It is, not the latest, but the direct, dominant, operative and efficient cause that must be regarded as proximate. If there are concurrent causes i.e. causes happening together and no excluded peril is involved there is liability under the policy. When an insured peril and an excepted peril operate together to produce the loss, the claim will be outside the scope of the policy.

If the results of the operation of the insured peril can be easily separated from the effects of the excluded peril, then there is liability under the policy. Where several events occur in unbroken sequence and no excepted peril is involved, the insurer is liable for all loss resulting from the insured peril. If an excepted peril precedes the happening of an insured peril, there is no claim If the insured peril is followed by an excepted peril, there is a valid claim for part, at least, of the loss. If the happening of an excepted peril is followed by the occurrence of an insured peril, as a new and independent cause, there is a valid claim for loss caused by the happening of an excepted peril. Modification of the doctrine: In the event of loss, the onus of proof [ or burden of proof] is on the insured. He has to prove that his loss is proximately caused by an insured peril. The onus is shifted to the insurer, if the insurer argues that the loss was caused by an excepted insurer will peril. They have to prove that the loss was proximately caused not by the insured peril, but by the excepted peril. Value of the doctrine: This doctrine serves not only to define the scope of coverage under the contract but serves also to protect the relative rights of the parties to the contract. It maintains a balance between the rights of the insureds and insurers. In the absence of this rule, every loss could be claimed by the insured and every loss could be rejected by the insurer It allows for the application of common sense to the interpretation of an insurance contract to the mutual advantage of the parties.

SECTION C CASE STUDIES


1. Many proponents feel that filling up of the proposal form is a boring and tiresome job and the agent of the insurance company should take care of that. But actually, that should not be the case. The proposer himself is supposed to fill up the same on his own. Despite that, many prefer the agent to do the job and most of the time the hapless agents do it without asking for any details of the health status of the proponent. This innocent act may run into a nightmare, if the policyholder dies within two years from the date of commencement of the policy. In such a situation, mandatory investigations are carried out before passing such claims and the insurer may reject the claim, if the probe reveals that the policyholder had withheld some information which was material information for the

purpose of underwriting by the insurer. It is the responsibility of the proponent to fill up the proposal form while the agent may only help him. Regulations too do not allow an agent to go beyond this. a. Despite a contract, how can the insurer repudiate its liability? b. How can you apply the concept of warranty in this context? c. How can you differentiate between a warranty and a representation? d. Discuss the legal authority of the insurer to repudiate its liabilities.

2. It is therefore the duty of the proposer to disclose, clearly and accurately, all material facts relating to the proposed insurance. Discuss the facts that are to be disclosed by the proponent as well as the facts that are not. 3. Krishna purchased an automobile service station from Venkata Rao. The purchase price included the building, equipment, and other assets. The business was financed by a loan from National Bank, which held a mortgage on the building. Krishna also converted a one-car repair bay into a short-order restaurant. When Krishna applied for property insurance on the business, he did not tell the insurance company about the restaurant because his premiums would have been substantially increased. Six months after the business opened, a car caught fire and damaged the roof over a bay in the service station area. a. Do any of the following parties have an insurable interest in the business at the time of the fire 1. Venkat Rao 2. Krishna 3. National Bank b. Venkat Rao told Krishna he could save money by taking over Venkat Raos insurance instead of buying a new policy. Would it be appropriate for Krishna to take over Venkat Raos insurance without notifying Venkat Raos insurer? c. Investigation of the fire revealed that the car owner knew the gas tank had a leak, but this information was not disclosed to Krishna when the car was brought in for service. Explain how subrogation might apply in this case. d. Did Krishna show utmost good faith when he applied for property insurance on the business? Explain.

e. Could Krishnas insurer deny coverage for the fire on the basis of a material concealment? Explain.

SECTION C ANSWERS
1. a. In this case, there is a high possibility that the agent may commit mistake and that may turn into a misrepresentation on the part of the policyholder. Moreover, the policyholder has to provide a warranty regarding the correctness of all the facts mentioned therein. Therefore, the contract will be treated as voidable at the option of the insurer owing to misrepresentation, if any thing wrong is written in the proposal form. b. A warranty is a promise collateral to the main contract, so that if there is a breach, it merely gives a right to damages and does not go to the root of the contract. In insurance contracts, however, a warranty corresponds to a condition in ordinary commercial law; that is to say, it entitles the party, aggrieved by the breaking of a warranty, to repudiate the contract either ab initio or from the date of the breach, according to the terms of the contract. A warranty must be complied with strictly and literally. It makes no difference whether the breach of warranty is material or immaterial, fraudulent or innocent, or whether the insurer would have been equally ready to insure at the same premium if aware of the breach. c. The differences between a representation and a warranty are as follows: i) A representation, even when material, need only be substantially correct, whereas a warranty must be strictly and literally complied with Before a misrepresentation can be held to be sufficient to enable the insurer to avoid the contract, it must be shown to be material to the risk. On the other hand, with a warranty, any breach is sufficient to avoid the contract, whether it is material or not. Whereas a representation does not appear in the policy, a warranty must be incorporated in the policy, whether expressly or by reference.

ii)

iii)

d. To protect the claimants and policyholders against any excessively harsh application of the duty of disclosure on the part of the proposer by the insurer, Section 45 of the Insurance Act, 1938, also called Indisputability or Incontestability Clause, has given the authority to the insurer to repudiate a claim in the following cases: The policyholder suppressed certain facts and those facts were material for the underwriting decision making by the insurer if any death claim arises within two years from the date of commencement of risk. But if any death claim occurs after two years from the date of commencement of the risk, an insurer may repudiate its liability to honor the claim under the following criteria: The policyholder suppressed certain facts and those facts were material for the underwriters decision making by the insurer and also those facts were intentionally suppressed by the insured. 2. The duty of disclosure must be observed throughout the negotiations and continues until they are completed and the contract is operative. The contract is deemed to be operative when the proposal is accepted by the insurer. This is called the condition of continued insurability which operates from the date of the proposal and the date of its acceptance. Material facts which are to be disclosed include the following: i. ii. Facts which tend to render a risk proposed greater than normal. Facts necessary to explain the exceptional nature of risk proposed for insurance where, without them, the insurer would justifiably believe the risk to be normal (Earlier proposal accepted with special conditions) Facts which appear to suggest some special motive for insurance e.g. gross over insurance (particulars of previous insurance to be disclosed). Facts which show that the proposer himself is in some way abnormal. E.g. revival may have been declined in an earlier policy.

iii.

iv.

In the absence of an enquiry, the following facts need not be disclosed. Consequently, they have no effect on the validity of the contract:

f. Facts which lessen the risk proposed for insurance g. Facts which could or should be inferred by the insurer in the wake of the particulars being actually disclosed. h. Facts of public knowledge, such as the existence of a state of war, or facts which should be known to the insurer in the ordinary course of his business. i. Matters of law. j. Facts with the possibility of discovery where the insurer has been given enough information to provoke enquiry on his part. The details furnished by the proposer in such circumstances must be adequate to fulfill his duty of disclosure.

k. Facts which can be reasonably concluded are a matter of indifference to the insurer, or regarding which he has waived information e.g., if the proposer inserts a disk in an answer to a proposal form question and the insurer makes no further enquiry. l. Facts which are superfluous to disclose by reason of a warranty in the proposed insurance.

3. a) Yes. Krishna is having insurable interest at the time of fire b) No, It is not appropriate for Krishna to take over Venkat Raos insurance without notifying Venkat Raos insurer c) The insurer of the car after compensating for the loss to the insured, can recover the loss from the Fire station owner / Krishnas insurer. d) No, Krishna did not show utmost good faith by not revealing material fact of that opening a short order restaurant in a car repair bay. e) Yes, Insurer can deny the coverage for the concealment of material fact by assuming that fire is due to restaurant operations.

CHAPTER 4 INSURANCE CONTRACTS


1.What is the minimum number of parties required for a legally valid contract? a. b. c. d. e. one Two Three. Four. Five

SECTION - A

2. Ram sends a proposal along with the requisite money to the New Vision Insurers seeking the required life insurance coverage for himself. Later on, he found that Horizon Insurers will be a better choice for him. Up to which point of time, he can cancel his proposal for insurance? a. b. c. d. e. Within one month from the date of dispatch of the proposal. Prior to the receipt of his proposal by the insurer Before the evaluation of his proposal by the insurer Before making any decision about his proposal by the insurer Before the acceptance of the proposal is communicated by the insurer.

3. When is the Communication of Acceptance by post treated as complete as against the proposer or the offeror? a. b. c. d. e. On receipt by the offerer. As soon as the letter is posted. Anytime during the transit of the letter. As soon as the letter of offer is accepted by the offeree. As soon as the letter is prepared by the offeree.

4. Which of the following renders an offer to be lapsed? a. b. c. d. e. Acceptance of the offer in the prescribed manner. Acceptance of the offer within the prescribed time limit. Through a counter offer by the offeree. By the revocation of the offer by the offeree. None of the above.

5. Which of the following is not an essential requirement of a legally valid contract? a. Free consent between the parties b. Competency to contract by the parties c. Lawful consideration d. Legal object. e. Expressly declared by the act to be void. 6. Which of the following leads to incapacity to contract? a. Minor age of any party. b. Undischarged insolvent. c. Citizen of an enemy country. d. Both (a) and (b) above e. All of (a), (b) and (c) above 7. Which of the following considerations is deemed as lawful? a. The consideration which is forbidden by law. b. The consideration which will defeat the provision of any other law of the land. c. The consideration which will impose some restrictions on both the parties. d. The consideration which will lead to some fraudulent activity. e. The consideration which is opposed to the public policy. 8. Which of the following is NOT a part of the policy document? a. The heading. b. The preamble c. The objective clause d. The operative clause e. The proviso. 9. What is the content of the preamble of a life insurance policy? a. The name and address of the insurance company. b. The intention of the insurer and the insured in brief and in general terms. c. The rights and responsibilities of the insured and the insurer under the insurance contract. d. The conditions to be followed and privileges of the insured for enjoying the insurance coverage. e. Stamps and signature of the authorized officer of the insurance company.

10. What is the content of the operative clause of life insurance policy? a. The name and address of the insurance company. b. The intention of the insurer and the insured in brief and in general terms. c. The obligations and responsibilities of the insured and the insurer under the insurance contract. d. The conditions to be followed and privileges of the insured for enjoying the insurance coverage. e. Stamps and signature of the authorized officer of the insurance company. 11. When will an insurance company be liable to pay the benefits, if the policyholder commits suicide? a. b. c. d. e. After three months from the policy date After six months from the policy date After six months from the date of commencement of the policy. After one year from the policy date. The risk of suicide cannot be covered.

12. In order to avoid the forfeiture clause, premiums for at least how many years are required to be paid by the policyholder? a. b. c. d. e. One year Two years. Three years. Four years. Five years

13. Which of the following is one of the duties of an insurance agent? a. To disclose the scales of commission in respect of the insurance product offered for sale, if asked by the prospect. b. To solicit or procure insurance business without holding a valid license. c. To include the prospect to omit any material information in the proposal form. d. To interfere with any proposal introduced by any other agent. e. To demand or receive a share of proceeds from the beneficiary under an insurance contract.

14. Payment of premium to an L.I.C. agent cannot be regarded as payment to the insurer because a. L.I.C. agents are not authorised to collect premium in accordance of rule 8 of Agents Regulation 1972 b. Satisfactory arrangements for payment of premium exist in L.I.C offices and hence payment of premium to an agent is not necessary c. L.I.C. feels the agent may misappropriate the premium d. None of the above 15. What is consideration on the part of the insured in respect of a life insurance contract a. A promise to take a new policy b. The payment made towards premium by the insured c. The insureds promise to pay the premium under the [policy d. Proposal completed and signed by the person seeking insurance is called consideration 16. The policyholders duty to disclose material facts lies at the time of a. b. c. d. 17. Taking a policy Revival of the policy Reinstatement of surrendered policy All the above

Which one of the following is considered an uninsurable peril a. Losses arising out of fire b. Losses arising out of war and war like operations c. Property risk d. Travel risks

18. According to the Marine Insurance Act 1963, a contract of marine insurance is valid a. Only when it is in writing b. Only when it is verbal c. Only if it fulfils the essentials of a valid contract d. It can be an oral agreement or a written contract

SECTION A ANSWERS
1. (b) For a legally valid contract, the minimum number of parties should be at least two the promisor who makes the promise and the promisee who receives the promise. Both sides of the contract should be aware of their respective considerations. 2. (e) In this case, Ram is offering money to the insurance company and thereby seeking insurance coverage from it. The contract between Ram and the insurance company will be established, if the offer is accepted by the insurer. However, the offer may be revoked by Ram anytime prior to the communication of its acceptance by the insurance company. 3. (b) As soon as the letter of acceptance is posted by the offeree, the communication with respect to the acceptance of the offered is deemed to be complete as per the legal requirement of a contract. 4. (c ) Making a counter offer by the offeree implies another offer to the offeror and the position of the offeror and the offeree is getting changed by that proposal. Therefore, a counter offer initiates another contract between the two parties while the earlier offer gets lapsed. 5. (e) If the contract is expressly declared by the Act as void, it cannot be called as a legally valid contract. 6. (e) Under the Indian Contract Act, 1872, a minor, an undischarged insolvent or the citizen of an enemy country are not eligible to sign a contract. 7. (c ) Any consideration that is forbidden by the law or defeated by the applicable regulations or is opposed to the public policy, cannot be considered as a legally acceptable contract. However, through a contract, both the parties may put some lawful restrictions on either side as part of the contractual benefits to be derived by them. 8. (c ) There is no part in a life insurance policy that is called as objective clause. 9. (b) The preamble of a life insurance policy describes briefly the intention of both the parties to the engagement of such a kind of contract. 10. (c ) In the operative clause, the responsibilities and obligations of both parties insured and insurer are mentioned. The insured is liable to pay

the premiums regularly while the insurer is obliged to disburse the secured benefits on the happening of the contracted event. 11. (d) In case of suicide by the policyholder within one year from the date of commencement of risk, the policy shall be void, otherwise the claim can be admitted. 12. (c ) As per the Insurance Act, 1938, a life insurance policy, whereunder the premiums are paid for a continuous period of three years, cannot wholly lapse but will acquire some guaranteed surrender value. This privilege is referred to as Non-forfeiture regulation. 13. (a) It should be done as per regulation 8 that specifies the code of conduct of an agent, holding a license of agency. 14. a 15. b 16. d 17. b 18. a

SECTION - B
1. . WHAT IS THE NATURE OF AN INSURANCE CONTRACT? Ans. The life insurance contract is having distinguishing features with the contract law. An offer and acceptance, legal capacity, valid consideration and a lawful agreement is sufficient for any contract to be valid. Apart from these requirements the insurance contract is having some more characteristics like adhesion conditional unilateral aleatory personal 2. GIVE A BRIEF NOTE ON Adhesion Contract Valued policy Unilateral contract Conditional contract Aleatory contract Personal contract

Ans. ADHESION CONTRACT: A contract of adhesion means the insured must accept the entire contract, with all its terms and conditions which are fixed by the insurer. It is highly specialized and technical in nature and prevents it from being a bargaining contract. VALUED POLICY: Unlike property insurance, life insurance contract is a valued policy which means the insurer agrees to pay a stated sum of money irrespective of the actual economic loss which is opposite to the contract of indemnity applicable to property insurance. UNILATERAL CONTRACT: Unlike commercial contracts which are bilateral in nature, insurance contracts are Unilateral in nature. It means that only one party, the insurer, gives a legally enforceable promise and the insured cannot be legally forced to pay the premiums. But of course, the insurer is bound to accept them and provide protection to the insured when they pay premiums in timely manner. CONDITIONAL CONTRACT: An insurance contract is a conditional contract. The insurer' s obligation to pay a claim depends upon the conditions which are inserted in the policy like payment of premiums, proof of death and in the case of property loss immediate notice of loss to the insurer etc. ALEATORY CONTRACT: Unlike a commutative contract, an insurance contract is aleatory. It means the values exchanged may not be equal but involves the element of chance or an uncertain event. In this one party may receive more in value than the other. The insured may receive more than the amount he paid in premiums when the loss occurs immediately after the acceptance of a policy. And the insurer may receive more in value if the loss does not occur to the insured when the policy was in force. PERSONAL CONTRACT: Property insurance is a personal contract between the insurer and the insured. In this, insurer insures the owner of property against loss but not the property. Insured property is indemnified if there is any loss and it cannot be assigned to another party without the insurer' s consent. But life insurance can be assigned to any one because the assignment does not alter the risk.

3. WHAT ARE THE BASIC REQUIREMENTS OF AN CONTRACT? Ans. The basic requirements of an insurance contract are Offer and acceptance. Consideration. Competent parties. Legal purpose.

INSURANCE

Offer and acceptance: An agent merely solicits or invites the prospective insured to make an offer. The applicant for insurance makes the offer, and the company accepts or rejects the offer. This accepting procedure is different for property, liability insurance and life insurance. In property insurance, the offer and acceptance can be oral or written and the agents have the power to bind their companies through the use of a binder which is a temporary contract. But in life insurance, the agent does not have the power to bind the insurer. The application for life insurance is always in writing and it should be approved by the insurer before it is in force. Consideration: It refers to the value that each party gives to the other. The insured' s consideration is payment or a promise to pay the premium and an agreement to abide by the conditions in the policy. The insurer' s consideration is the promise to do certain things which are specified in the contract. Competent parties: Insane persons, intoxicated persons and minors are not legally competent to enter into insurance contracts. Legal purpose: All illegal activities which are contrary to the public interest are not enforceable. A contract should have a legal purpose 4. DESCRIBE THE BASIC PARTS OF AN INSURANCE CONTRACT: Ans. Insurance contracts generally can be divided into Declarations Definitions Insuring agreement /operative clause Exclusions Conditions Miscellaneous provisions

Generally most of the insurance contracts contain these parts. DECLARATIONS: In the declaration part information regarding property or activity to be insured by the insurer is presented. It is useful for underwriting and rating purposes. It is generally found on the first page of the policy. In property insurance, the declaration page contains information regarding the identification of the insurer, name of the insured, location of the property, period of protection, amount of insurance, amount of the premium etc. In life insurance, the declaration page contains the insured' s name, age, premium amount, issue date and policy number. DEFINITIONS: Insurance contracts generally contain a page of definitions. They are in either quotation marks or in bold face type. For ex. The insurer is always referred to as "we" "our" or "us" and the insured is referred to as "you" and "your". These definitions make easy to determine the coverage under the policy. INSURING AGREEMENT: It is the most important part in the contact. It summarizes the major promises and conditions to fulfil the promises by the insurer. In property and liability insurance there are two forms of an agreement. 1. Named peril policy and 2 all risk coverage. In the named peril policy the policy covers only the specified perils, whereas under an all risk coverage policy the policy covers all losses except those losses specifically excluded. All risks coverage is preferable to named perils coverage because of its broad coverage and also as the burden of proof to deny the payment is placed on the insurer unlike the insured in the named peril policy. Life insurance is an example of all risks policy - it covers all causes of death with some exceptions like suicide, aviation, war etc. EXCLUSIONS: There are three types of exclusions in an insurance contract. a. Excluded perils [ ex. The peril of war is excluded in disability income policy] b. Excluded losses [ ex. Professional liability losses are excluded from personal liability section of home owners policy]. c. Excluded property [ ex. Cars and planes in a home owners policy] Main REASONS for these exclusions are Some perils are uninsurable like wars. Some perils are predictable declines [wear and tear and inherent vice] Some perils are extraordinary hazards [in cars and cabs - cabs are more prone to accidents than car.]

To avoid duplication of coverage [ ex. A car is excluded under home owners policy because car is covered under the personal auto policy.] To avoid moral hazard - If unlimited amount of money were covered fradulent claims could increase. Finally exclusions are used because the coverage is not needed by the typical insured. CONDITIONS: To perform promises by the insurer the conditions section imposes certain duties on the insured. If the policy conditions are not met, the insurer can refuse to pay the claim. MISCELLANEOUS PROVISIONS: In property and liability insurance, miscellaneous provisions refer to cancellation, subrogation, assignment of the policy. And in life insurance grace period, reinstatement of a lapsed policy and misstatement of age etc. Most of the policy owners make a common mistake of not reading policies and understanding the contractual provisions that appear in the policies which lead to controversies in the settlement. To avoid this, study of basic parts of insurance contract would be useful. 5. Identify the major types of exclusions typically found in insurance contracts. Why are exclusions used by Insurers? Ans. EXCLUSIONS are basic part of any insurance contract. There are three types of exclusions in the insurance contract. a. Excluded perils [ ex. The peril of war is excluded in disability income policy] b. Excluded losses [ ex. Professional liability losses are excluded from personal liability section of home owners policy]. c. Excluded property [ ex. Cars and planes in a home owners policy] Main REASONS for these exclusions are Some perils are uninsurable like wars. Some perils are predictable declines [wear and tear and inherent vice] Some perils are extraordinary hazards [in cars and cabs - cabs are more prone to accidents than car.] To avoid duplication of coverage [ ex. A car is excluded under home owners policy because car is covered under the personal auto policy.]

To avoid moral hazard - If unlimited amount of money were covered, fradulent claims could increase. Finally exclusions are used because the coverage is not needed by the typical insured. significance of the conditions section to the insured? Ans. The conditions section is an important part of an insurance contract. The CONDITIONS are provisions in the policy that qualify or place limitations on the insurer' s promise to perform. So the conditions section imposes certain duties on the insured. If the policy conditions are not met, the insurer can refuse to pay the claim. 7. How do you construct an insurance contract? Ans. Generally speaking, the construction of the insurance contract as to its validity will be governed by the law and usages of the place where the contract is made. Like any other legal contract, the construction of an insurance contract also depends on the agreement between competent parties, resulting from an offer and acceptance, must comply with legal requirements as to form and have for its purpose a legal object. There must be a valuable consideration, an insurable interest and the contract must be made with full knowledge of material facts and be free from fraud, mistake and misunderstanding. And policy forms are necessarily general in character, since they are drawn to meet a general situation and not with reference to particular cases. But because of ambiguity in the wording and varying circumstances surrounding the loss, the chance of disputes as to the interpretation may occur. To avoid this there are certain legal principles underlying the interpretation of the application, as well as of policy provisions, that are kept in mind by the courts as guiding principles in their efforts to interpret the contract. Ex. 1. Courts consider fire insurance is a contract of indemnity where as life insurance is not a contract of indemnity though the principle of indemnity is a fundamental legal principle of an insurance contract. 2. In the case of contract of adhesion and endorsement, generally the courts will give the benefit of the doubt to the insured on the ground that the contract is not a bargaining contract. 3. Finally, it is the purpose of the law and function of the court to enforce the contract when there is no conflict or ambiguity between the two competent parties insured and the insurer. 6. Define the term conditions in an insurance contract. What is the

8. Describe the schedule of an insurance policy. Ans. Life insurance policies are generally two types Narration type and Scheduled type In the NARRATION TYPE the contract contains continuous details in a long narration which is not popular in modern times. SCHEDULED TYPE is popular in modern times. It contains Date of execution or operation Parties names Preamble or recitals [object of the deed] Operative clause or consideration [ insurer' s promise to pay the sum assured to the insured on the condition of scheduled payments] Covenants of title and Conditions [covenants like a. right to convey, b. for freedom from encumbrances, c. for further assurance and conditions like declaration of age insurable interest, valuable consideration etc.] Schedule or parcels [ accurate description of the property in property insurance and Benefit Schedule [ particulars of the assured, the event insured, the sum assured, the payees etc..] and payable schedule [ particulars of the payment of dates of first premium, renewal premium etc..] in life insurance.] Habendum [the quantity of the interest or estate of the assignee]. Execution [ prescribed articles for execution of the deed]. Attestation of signature. Delivery -- unless a deed is delivered it will not be operative. There should be a great care in drafting the policy. There should not be any ambiguity in the wording and it should reflect the intention of the both the parties. 9. What is an Endorsement or rider? If an endorsement conflicts with a policy provision, how is the problem resolved? Ans . Insurance contracts generally contain endorsements and riders. The terms endorsements and riders are interchangeable. Endorsements in property and liability insurance modify, extend or delete provisions found in the original policy. In life and health insurance, riders can be added that increase or decrease benefits, waive a condition of coverage present in the original policy or amend the basic policy. If an endorsement is contrary to a law or regulation the policy is read and applied as if that endorsement did not exist.

10. Why do deductibles appear in insurance contracts? Identify some deductibles that are found in insurance contracts. Ans. This provision is generally found in property, health and auto insurance contracts and not in life insurance contracts because insured' s death is a total loss and deductibles reduce the face amount of insurance. According to Rejda, a deductible is a provision by which a specified amount is subtracted from the total loss payment that otherwise would be payable. Main purposes of these deductibles are To eliminate small claims [expensive to handle and process]. To reduce premiums[ larger deductibles are preferable to smaller deductibles]. To reduce moral and morale hazard [ because of it insured may not profit out of loss and it encourages insured to be careful of his property] Deductibles in property insurance are two types. straight deductibles [applies to each loss] aggregate deductibles [ all covered losses are added together till they reach a certain level for a particular period.] Deductibles in health insurance generally are calendar year deductible [aggregate]. corridor deductible [ it is a supplement plan to major medical expense plan]. elimination [waiting] period deductibles. [disability income contract for the loss of work earnings]. 11. WHAT IS COINSURANCE? Ans. It is a contractual provision in commercial property insurance. Under this clause the insured must insure the property for a stated percentage of its insurable value [actual cash value or replacement cost ] to collect in full for a partial loss. If this coinsurance is not met at the time of loss, the insured must share in the loss as a co insurer. The co insurance formula is Amount of insurance carried ------------------------------------- x loss = amount of recovery Amount of insurance required

Amount of insurance required is the actual cash value of the property x % coinsurance clause in the policy.

The precaution in applying this coinsurance formula is that the amount paid can never exceed the amount of the actual loss and should be limited to the face amount of the insurance. The main PURPOSE of co insurance is to achieve equity in rating [ the insured who met co insurance receives a rate discount and who is under-insured is penalized through application of the coinsurance formula] But practical PROBLEMS IN OPERATING CO INSURANCE clause arise from changes in the actual cash value of the property due to inflation or with increase in inventory values with increase in stock etc. 12.Describe a typical co insurance clause [ percentage participation clause ] in a major medical policy? Ans. Co Insurance In Health Insurance is technically known as Percentage Participation Clause. Under this the insured has to pay a certain percentage of medical expenses in excess of the deductibles. The PURPOSE is to introduce this participation is 1. To reduce premiums, and 2. To prevent overutilization of policy benefits like expensive medical services.

SECTION C CASE STUDIES


1. Joshua owns a motorboat that struck and damaged another boat anchored at a marina. When the accident occurred, Joshuas friend, Zoe, was operating the motorboat. The owner of the damaged boat has asked Zoe to pay for the damage. Joshua is listed as the named insured on the declarations page of a boatowners liability policy. Joshuas policy included the following provisions: What is covered? We will pay all sums which an insured becomes legally obligated to pay as damages due to bodily injury or property damage arising out of the ownership, maintenance, or use of the described vessel. What Is Not Covered We do not cover liability arising from the ownership, maintenance, or use of the described vessel: 1. that results from an intentional act of an insured

2. to persons while they are being towed by the described vessel as water skiers, aquaplaners, or similar water sports. Definitions 1. The words you and your mean the persons named on the Declarations. 2. The words we, us, and our mean the company providing this insurance. 3. Bodily Injury means bodily harm, sickness, or disease to a person. 4. Insured means you and: your spouse; your relatives if residents of your household; persons under the age of 21 in your care; a person who operates a covered vessel with your permission. Property damage means an injury to or the destruction of property. This includes the loss of use. The above provisions refer to a described vessel. Identify the section of the policy that would describe Joshuas motorboat. Explain whether Joshuas policy would provide coverage for the claim against Zoe. Identify the policy provisions that would be relevant in this situation. The owner of the damaged boat uses the boat to transport customers for deepsea fishing. The owner is also holding Zoe responsible for the loss of revenues while the boat is being repaired. Explain whether Joshuas policy would pay for the lost revenues. Assume that a skier towed by Joshuas motorboat is injured because of Joshuas negligence. If the injured skier sued Joshua for the injury, would Joshua be covered under the policy? Explain.

a. b.

c.

d.

1. a) Declaration

SECTION C ANSWERS

b) Joshuas Policy can provide coverage for the claim against Zoe. The policy provision in this situation is that under the Definitions a person who operates a covered vessel with your permission is also included.

c) Yes, Joshuas policy would pay for the lost revenues because under the provision what is covered the damage arising out of the use of described vessel is also covered. d) No, because skiers are not covered under the provision what is not covered.

CHAPTER 5 MANAGEMENT OF INSURANCE ORGANIZATIONS


SECTION - A
1. Insurance institutions are basically a. b. c. d. Financial institutions Investment institutions Mutual funds institutions Lending institutions

2. The regulator of insurance companies in India a. SEBI b. IRDA c. LPA d. TAC 3. The authority responsible in the redressal of disputes between the insurer and the insured is known as a. Ombudsman b. Arbitrator c. Judge d. advocate 4. Ombudsman is a a. Governmental body b. Semi-governmental body c. Semi-quasi body d. Semi-judicial body

5. The insurance ombudsman pronounces a. statements b. decree c. awards d. rewards

6. The TAC in India is mainly concerned with the a. b. c. d. regulating of rates issuance of policies investments of the funds licensing of the agents

7. which of the following is not a part of the underwriting process a. Selecting policy holders b. Determining markets for insurance products c. Determining the insurers selection criteria d. Supervising the rate process 8. If an insurer charges low premium rate, he will gain a competitive advantage through a. production efficiencies b. risk -selection process c. adverse selection d. both a and b 9. the acceptance of substandard exposure in return for other more profitable business is generally described as a. underwriting audit b. underwriting guide c. policy statement d. accommodation

10. which of the following is referred to as general insurance a. b. c. d. public insurance property insurance private insurance personal insurance

11. The broker in an insurance business is the representative of a a. b. c. d. agent insurer insured principal

12. An individual experience modification program that uses the current year as the experience period to develop the experience modification factor is called a. schedule rating b. retrospective rating c. experience rating d. policy rating 13. The gross premium includes besides risk premium a percentage of loading factor for a. b. c. d. Expenses past losses adverse claims experience profits

14. the process of evaluating risks and fixing rates in an insurance company is called a. b. c. d. underwriting rate making under rating reinsurance

15. which of the following measures are intended to lower frequency of losses a. loss prevention measures b. loss control measures c. loss reduction measures d. loss minimization measures 16. The process of insuring losses for an entire class, territory , or book of business is known as a. b. c. d. treaty reinsurance portfolio reinsurance facultative reinsurance proportional reinsurance

17. excess of loss treaties can be classified as a. b. c. d. per occurrence per risk aggregate excess all the above

18. which of the following is the most effective form of reinsurance for handling loss frequency variations a. aggregate excess treaty b. facultative treaty c. catastrophe treaty d. pro rata treaty 19. When a reinsurer shares his loss exposure(s) with other reinsurer(s), it is called as a. reinsurance b. retrocession c. retention d. re-reinsurance 20. who among the following does not come under the category of claims adjusting personnel a. assessor b. examiner c. claims supervisor d. adjuster 1. a 2. b 3. a 4. c 5. c 6. a 7. d 8. d 9. d 10. b 11. c 12. b SECTION A ANSWERS 13. a 14. b 15. a 16. b 17. d 18. a 19. b 20.a

SECTION - B
1. Underwriting is said to be the heart of insurance operations. Explain the factors that are considered by insurance companies when underwriting vehicle exposures? Ans : Generally, insurers measure the desirability of a vehicle by analyzing the characteristics of the insured drivers. Many systems are used for evaluating the loss potential of private as well as commercial automobile applicants. Some of the major underwriting factors that are considered for evaluating the loss potential are: i) ii) iii) iv) v) vi) vii) viii) ix) x) Age of operators Age and type of automobile Use of the automobile Driving record Territory or radius of operation Sex and marital status Occupation Personal characteristics Physical condition Safety equipment.

Age of operators Usually, considerably higher rates are charged for young drivers. This is due to the greater involvement of young drivers in fatal accidents. Age and type of automobile A correlation exists between the age of the vehicle and its mechanical condition. The type of automobile may also determine underwriting acceptability. Sports and luxury cars tend to produce higher losses than other models. Use of the automobile Underwriting of automobiles also takes into account the type of roads the automobile runs on (highways or city roads,) the distance covered by it on a daily basis, and the purpose for which the automobile is used (for business use, for pleasure, etc.) Driving record The record or prior loss history of the driver is taken into account when evaluating a private passenger automobile applicant.

Territory or radius of operation Accident frequency depends not only on the territory in which the automobile is used but also on the distance traveled by the automobile. Sex and marital status Young female drivers are considered to be more careful drivers than young male drivers, and are therefore considered better than male drivers in terms of loss exposure. Occupation Some occupations require extensive driving. increases the probability of loss. This

Personal characteristics The credit information of applicants is often used by insurers to evaluate their financial stability. Underwriters have discovered a correlation between poor credit risk and poor insurance risk. 2. What are the risk dimensions of underwriting in the insurance business as opposed to the securities business? Ans. Investors seek diversification of risks and commensurate market rates of return. Otherwise, it may lead to pile up of high price/earnings (P/E) multiple stocks. Therefore, generally, investors try to maintain a balance between the real risk and the perceived risk and desire to have a balance of higher and lower P/E stocks. P/E ratio measure risk adjusted rates of return. They indicate the degree of risk as well as the expectation of earnings and cash flows in the near term. Buyers of insurance want a stable and credit quality market, which provides them with a reasonable ability to forecast and budget related costs. The risk involved in the insurance business is the unexpected or fortuitous causes of loss defined in the policies. Underwriting firms, on a portfolio basis, focus on matching assets and liabilities. However, they maintain a reasonable degree of liquidity in order to pay claims. Further, in the insurance business, the insurer got the responsibility towards its policyholders apart from the investment philosophy of maintaining safe highcredit quality (lower yields) fixed income investments. Generally, insurers maintain a rate of return between the 6 to 10 percent. Insurers more or less abide by the regulations of the regulatory authorities to avoid losses. As per the regulations, they hold stocks in their investment portfolios.

In the securities business investors seek a balance of risk and slight tilt towards high risk and except an array of returns. The capital market firms proclaims about the high returns in the market. But this is possible because their portfolios would have large inventories of higher risk investments. For example, there is big correction in the market in recent times with the bursting of Information Technology bubble. Therefore, those who invested heavily in such stocks are now correcting their book values to the lower of cost or market as required by generally accepted accounting principles (GAAP). 3. A majority of insurance claims pertain to losses caused by fire. Underwriting of natural disasters also constitutes a major portion of insurance activities. What are the possible losses that can arise due to disasters? Suggest a broad framework for disaster management in order to minimize such losses? Ans : A natural disaster is defined as any natural phenomenon, which causes such widespread losses to lives, property or the environment, from which the affected population cannot recover without external assistance. Cyclones, floods, storms, earthquakes, drought, forest fire, avalanches, etc are some examples of natural disasters. Disasters are mostly natural but they can also be caused by man. The world is prone to natural disasters which are caused by the interplay of natural forces and human activities. A broad categorization of losses that arise due to natural disasters are: Direct losses: These losses are in the form of damage to buildings, its contents, infrastructure, loss of human lives, costs of clearing the debris, restoration, loss mitigation and disposal. Indirect losses: These are losses that result due to business interruption and power failure, costs of transportation, detours, assistance, storage, accommodation, provision of essentials, and communications. Intangible losses: These include psychological trauma and impairments, losses of intangible values, and losses due to evacuation from areas under risk. A risk management plan consists of the following steps:

Risk Identification: It involves identifying the extent and intensify of past events, areas that are hit by one event at the same time, potential risk areas, and climatic trends. Risk Evaluation: It involves assessing the probability of the occurrence of a disaster and the severity of the disaster. Risk Control: This step covers measures aimed at avoiding, eliminating or reducing the chances of occurrence of loss-producing events and eliminating the severity of the disasters that could happen. Risk Retention: This step pertains to the ability of the country or the society in particular to withstand disasters. There are two aspects of risk retention psychological and financial. The psychological aspects have a lot to do with the involvement of government machinery and voluntary agencies and also to a great extent on the tenacity of the people. Risk Transfer: This is a risk financing method and provides a means for handling risk which have a high severity of losses and one cannot afford to retain such risks. 4. What are he parameters to decide whether a particular risk is insurable or not? Ans. The key parameters to decide whether a particular risk is insurable or not are: Mutuality: A large number of people exposed to risk must combine to form a risk community. Need: The occurrence of the anticipated event must place the insured in a condition of financial need. Assessability: The expected loss burden must be assessable. Randomness: The time at which the event occurs must not be predictable and the occurrence of the event must be independent of the will of the insured. Economic Viability: The community organized by the insured persons must be able to meet its future loss-related financial needs on a planned basis.

Similarity of threat: The insured community must be exposed to the same threat. Further, the occurrence of the anticipated event must create the need for funds in the same way for all those affected. 5. In reinsurance transactions, what does retention mean? What are the steps being considered by the Insurance Regulatory and Development Authority to improve the retention capacity of domestic insurance companies? Ans. Reinsurance agreements involve a transfer of loss exposures assumed by the primary insurer to the reinsurer. However, the reinsurer does not assume all the exposures to which the primary insurer is exposed. The reinsurance agreement requires that a portion of the exposure, known as retention, be retained by the primary insurer. Domesic insurance companies are facing low retention capacities owing to the increasing outflow of their premium income from the country. The outflow of premium is a result of the large amounts of reinsurance taken by private insurance companies, either with their parent company abroad or with other large reinsurance companies. Private sector insurance companies have retained only a small part of the risk that they have insured and reinsured a major portion of the risk insured owing to under-capitalization. Further, reinsurers have hiked their premiums to close to one percent of the sum assured. With this, the low retention capacity of the private insurance companies will lead to an outflow of premiums to the extent of 40 percent of the gross domestic premium income of all general insurance companies. In contrast to the insurance companies in the private sector, insurance companies in the public sector have seldom taken large reinsurance cover. Even in the worst cyclical phase, the reinsurance taken by the four public sector insurance companies has hardly exceeded 7 percent of the gross domestic premium income. This has been possible because of the high retention capacity of public sector insurance companies with each company having an average capitalization of about Rs. 2,500 Crore. Thus, in order to cap premium outflows from the country and improve the retention capacity of domestic insurance companies, the Insurance Regulatory and Development Authority is considering the following measures:

Coinsurance It is an arrangement between all domestic insurance companies identical to consortium financing by banks. In coinsurance, there will be one lead insurer with several other insurance companies. All the member companies in the arrangement will have a share in premium collections. They will share the claims on a pari passu basis (pari passu fairely; at an equal rate or in an otherwise fair way, with no person or group taking percent over another) Syndicated insurance This is identical to coinsurance arrangement, except that the members of the syndicate have the option to exit the agreement. Increased Capitalization Increasing the capitalization of the new private sector companies in the general insurance industry is likely to improve the retention capacity of private insurers. 6. Discuss the objectives of ratemaking activities in an insurance organization. Ans: Ratemaking is one of the functions of actuaries. The basic objective of ratemaking is to develop a rate structure that will enable the insurer to compete effectively for business while earning a reasonable profit. Apart from this primary objective, there are certain secondary objectives. They are. Corporate objectives Regulatory objectives Corporate objectives - Primary insurers prefer rates that are flexible, easy to apply, promote loss control and provide for contingencies. However, in reality, this is not possible to achieve. Primary insurers prefer stable rates because the process of fixing a rate is very expensive. In addition, frequent changes would lead to customer dissatisfaction.

To promote loss control, the rating system should provide lower rates to those policyholders who undertake loss prevention measures. Apart from this, the rating system should be understandable to agents, brokers, underwriters and especially policyholders (who have little expertise regarding insurance matters).

Regulatory objectives According to regulatory objectives, rates Should be adequate Should not be excessive Should not be unfairly discriminatory If the rate does not comply with regulatory norms, it may invite harsh regulatory actions and unfavorable legislation. 7. A property-liability insurer basically operates two kinds of businesses. Comment on this statement and also explain the role of investment income in ratemaking. Ans. A property liability insurer basically operates two kinds of businesses: Insurance business Investment business Insurance business Insurance business operations include writing policies, collecting premiums, and paying claims. This is the revenue generator for an insurance company. The success of an insurer depends on the success of his insurance business. Investment business This business includes the purchase of bonds, stocks and other investment vehicles for earning profit. In the investment business, profit is derived from three sources: investment income, realized capital gains, and unrealized capital gains. Role of investment income in ratemaking Previously, investment income was not considered by insurers when calculating insurance rates. But, off late, in some countries, it is considered in the calculation of rates. Investment profits can be reflected in ratemaking by first estimating the realizable investment profit from a particular insurance line, and later deducting that amount from the expense loading used in the rates. The inclusion of investment profits in ratemaking may affect liability insurance rates more than property insurance rates because the amount of investment profit earned on an insurance line would largely depend on the loss reserve and the unearned premium reserves.

SECTION C CASE STUDIES 1. Rahul Kumar, the CEO, Ramesh Adhikari, the General Manager (Underwriting), of RK Insurance Company are discussing the various types of reinsurance options available in the market. Adhikari informed Kumar that three types of reinsurance options are available: facultative, treaty and financial reinsurance. He also described each option briefly to the CEO. After listening to him, the CEO said that, prima-facie, facultative reinsurance seems to be more suitable for the company than other types of reinsurance. He asked Adhikari to provide him details on the different types of facultative reinsurance available. Describe the different types of facultative reinsurance contracts available to help Adhikari make a successful presentation. Ans: In facultative reinsurance, the loss exposure is written separately for each individual customer by the reinsurer. The primary insurer provides the reinsurer detailed information about each exposure. The facultative reinsurer is not bound by the rates quoted or charged by the primary insurer. When the reinsurer accepts a particular exposure, he issues a certificate of reinsurance to formalize the agreement. Even though, in the case of facultative reinsurance, reinsurers have the right to change rates and charge higher premium, primary insurers still opt for it. Facultative reinsurance can be used by the primary insurer to protect its treaties, protect a favorable commission under its treaties, or protect a profit-sharing agreement. Apart from this, a facultative agreement is an independent transaction and is underwritten separately. Consequently, a loss under one facultative agreement will have little or no affect on either the terms or the rates of other subsequent transactions. There are two categories of facultative reinsurance: Pro rata facultative reinsurance Excess facultative reinsurance Pro rate facultative reinsurane Facultative reinsurance for property exposures has traditionally been written on a pro rata basis. It is similar to a surplus share treaty, except that facultative agreements relate to a single subject of insurance. Excess facultative reinsurance In this type of reinsurance contract, the reinsurer is responsible only for those losses that exceed the insurers

retention, and it pays only the amount in excess of the retention, upto reinsurance limit. 2. An insurer has 10,00,000 earned car-years and Rs. 4,000 of incurred losses. The cost for issuing a policy and collecting the premium would be Rs. 100 per car-year. The other underwriting expenses vary with the size of the premium and total upto 20 percent of the gross premium. Calculate the gross rate using the pure premium method. (Note: Assume that the fixed cost does not change with the size of the premium, the rating class, or the rating territory). Ans: The pure premium method consist of three steps: Calculating the pure premium Calculating the expense loading Combining the pure premium and the expense loading into the gross rate The pure premium is calculated by dividing the number of earned exposure units by the amount of incurred losses. So, it is 10,00,000/4000 Rs. 250 per car-year. So, the gross rate is G = (P + F) 1-V) Where G stands for gross premium P stands for pure premium F stands for amount of fixed expenses V stands for percentage of variable expenses G = 250 +100 (1 0.20) So, G = 350/0.80 This gives G as 430.75. The gross premium in the above case is Rs. 430.75

3. Of the required coverage amounting to Rs. 1,80,000 a policyholder maintained only Rs. 1,20,000. The deductible amount was Rs. 10,000. Which of the two methods deductible applied first and coinsurance applied first is favourable to the insured in deciding the settlement amount for a loss amounting to Rs. 30,000? Ans: Deductible Applied First: Loss Deductible = Amount payable to the policyholder Rs. 30,000 Rs. 10,000 = Rs. 20,000 (Coverage actually maintained by the policyholder coverage required to be maintained) x Amount payable to the policyholder = Recoverable amount of loss. (Rs. 1,20,000 / Rs. 1,80,000) x Rs. 20,000 = Rs. 13,333.33. Coinsurance Applied First: (Coverage actually maintained by the policyholder Coverage required to be maintained) x Loss (Rs. 1,20,000/Rs. 1,80,000) x Rs. 30,000 = Rs. 20,000 Recoverable amount of loss is Rs. 20,000 Rs. 10,000 = Rs. 10,000 Therefore, by comparing the recoverable amount of loss obtained by the two methods, it is evident that the deductible applied first method is more favourable to the insured since he receives a higher recoverable amount of loss.

CHAPTER 6 GENERAL INSURANCE POLICIES (PERSONAL)


SECTION - A
1. Personal accident policies cover the risk of a. unemployment b. sickness / illness c. accidental injuries d. none of the above 2. Personal Accident policies are a. valued policies b. indemnity policies c. unvalued policies d. open policies e. none of the above 3. Disability income policies pay for a. disability arising out of sickness b. disability arising out of sickness / accident c. disability arising out of employment injuries d. hospitalization only for injuries e. none of the above 4. Personal liability insurance policies cover damages to properties and injuries to other people due to a. Negligence b. willful misconduct c. war losses d. burglary e. none of the above 5. The fire insurance policy in India does not provide for compensation for losses arising out of a. Theft b. aircraft damage c. implosion or explosion d. lightning e. none of the above

6. Workmens Compensation policy indemnifies the losses of the a. worker b. employer c. government d. surveyor e. none of the above

7. golfers indemnity policy provides protection against losses or damages to the a. only golf players b. golf players & golf equipment c. only golf equipment d. golf players & golf equipment & liability losses e. none of the above

SECTION A ANSWERS
(1) c (2) a (3) b (4) a (5) a (6) b (7) d

PERSONAL ACCIDENT POLICY.

SECTION - B

1. Define Accident as given in the Personal Accident policy. Ans: Accident is defined as any injury caused by accidental, violent, external, and visible means resulting in death or disablement. 2. Is the Personal Accident insurance, a contract of Indemnity? Give reasons for your answer. Ans: No, it is not a contract of indemnity because on some conditions being fulfilled the Capital Sum Insured (CSI) is payable to the injured. For example, if within twelve months of its occurrence the injury be the sole cause of death, or of total and irrecoverable loss of both the eyes, or loss of use of two hands or two feet, the capital sum insured shall be payable to the insured. Thus, the contract is based on agreed value of loss. Different compensations for different classes of insured negate the principal of indemnity.

3. How is the Sum Insured or Limit of Indemnity fixed? Ans: The sum insured is usually fixed in multiples of the annual earnings of the insured, for example, at least five times the annual income in most cases. For Group / Personal Accident Insurance, sometimes two years annual salary is taken. 4. Can a person recover anything under a Personal Accident policy, if he dies a natural death? Ans: No, if the insured dies a natural death due to disease or old age or due to his intentional act like suicide, or even when death results due to War, Seizure, Capture, Mutiny and so on, he is not entitled to recover anything under the policy. 5. MENTION THE CONDITIONS OR SITUATIONS WHEN ONLY 50% OF THE CSI IS PAYABLE IN THE PA COVER? Ans: If the injury to the insured shall within 12 calendar months of its occurrence be the sole and direct cause of the total and irrecoverable loss of- the sight of one eye or of the actual loss by physical separation of one hand or one entire foot - total and irrecoverable loss of one hand or one foot without physical separation, 50 % of the CSI is payable to the insured. 6. How is compensation for loss of capacity assessed with the help of the schedule? Ans: If the injury shall immediately, permanently, totally, and absolutely disable the Insured person from engaging in any employment or occupation of any description, then a lumpsum equal to 100 % of the CSI stated in the schedule is payable to the insured. If the injury shall be the cause of the total and irrecoverable loss of use or of the actual loss of any other part of the body, then a percentage of the CSI as indicated in the schedule is payable to the insured.

7. What is the compensation payable in case of temporary total disablement in a PA cover? Ans: If the injury shall be the sole and direct cause of temporary total disablement, then so long as the insured is disabled in any employment or occupation a sum at the rate of one percent (1%) of the CSI as stated in the schedule is paid per week, but in any case not exceeding Rs.5, 000/- or 25% of the monthly salary whichever is lower for a period not exceeding 100 weeks in respect of any one injury. The schedule may differ from insurer to insurer. 8. What is the liability of the Insurance Company, when the insured dies outside his/ her residence under a PA cover? Ans: In the event of death of the Insured Person due to accident as defined in the policy outside his/her residence, the company shall reimburse expenses incurred for transportation of Insureds dead body to the place of residence subject to a maximum of 2 % of CSI or Rs.1,000 whichever is less. This will be in additional to any other compensation for death payable under the policy. 9. How is Cumulative Bonus determined under a Personal Accident cover? Ans: The compensation payable under the policy for death, loss of limb(s), or sight and for permanent Total Disablement arising out of accidental injuries shall be increased by 5 % in respect of each completed year, during which the policy is in force, prior to the occurrence of an accident for which CSI shall be payable and if no claim has been made earlier. 10. What are the exceptions under a PA policy? Ans: The Personal Accident policies contain exceptions like death, injury or disablement, directly or indirectly caused by, arising out of, or resulting from, or traceable to a) Intentional self injury, suicide, insanity or influence of intoxicating drinks or drugs or b) In the case of women, by childbirth, pregnancy, etc. c) Whilst engaging in aviation or ballooning. d) Whilst mounting or dismounting from a balloon or aircraft other than as a passenger e) War, invasion, act of foreign enemy, hostilities, civil war, rebellion, revolution, seizure, capture, arrests, restraints and detainment.

f) Ionizing radiation, contamination from combustion of nuclear fission. HEALTH INSURANCE 11. Describe the salient features of the Individual Mediclaim insurance policy. Ans: The individual Mediclaim policy covers reimbursement of Hospitalization / Domiciliary Hospitalization expenses for illness/ diseases or injury sustained. The company will pay to the Insured Person the amount of such expenses as are reasonably and necessarily incurred but not exceeding the Sum Insured in any one period of insurance. 12. What is the scope of cover under the various clauses in a health insurance policy? Ans: In the event of any claim/s becoming admissible under the scheme the company will pay to the insured such expenses as mentioned below: - Room expenses, boarding expenses as provided by the hospitalization/ nursing home. - Nursing expenses - Surgeon, anesthetist, medical practitioner, consultants and specialists fees. - Other expenses relating to blood, oxygen, surgical expenses. 13. What Institutions can be considered as Hospital/ Nursing Home under a Mediclaim policy? Ans: Hospital/ Nursing Home means an institution in India established for indoor care and treatment of sickness and injuries and which should have been registered with local authorities and should be under the supervision of a qualified medical practitioner or should comply with minimum criteria of having 15 in-patients beds, fully equipped operation theatre and with fully qualified nursing staff and doctors round the clock. 14. What are the benefits under a Domiciliary Hospitalization cover of health insurance? Ans: Medical treatment exceeding three days for such illness /disease/ injury which in the normal course would require care and treatment at a hospital/ nursing home but actually taken at home in India is referred to as Domiciliary hospitalization. The benefits can be availed under the following conditions, namely-

- the patient cannot be removed to the hospital due to his/her condition - there is lack of accommodation in the said hospital/ nursing home. 15. What is considered as Surgical Reimbursement under the policy? Operation for the purpose of

Ans: Surgical operation means any manual and /or operative procedure for correction of deformities and defects, repair of injuries, diagnosis and cure of diseases, relief of suffering and prolongation of life. 16. Who is a Qualified Nurse? Ans: Qualified Nurse means a person who holds a certificate of a recognized Nursing Council and who is employed on the recommendations of the Medical Practitioner. 17. Mention the major basic coverage provided in an individual Health insurance. Ans: Health insurers sell a wide variety of individual health insurance coverages. Important individual coverages include the following: Hospital surgical insurance Major medical insurance Long term care insurance Disability income insurance 18. What is a Basic Health Plan cover? Ans: It is principally a hospital- surgical insurance and it takes care of routine medical expenses as well. However it does not pay for catastrophic losses.

19. What is a Complete Plan under a health insurance cover? Ans: Under a Complete Plan, the following are coveredHospital expenses Surgical expenses Out- patient, diagnostic, x-ray and laboratory expenses Physicians in-hospital expenses Maternity expenses 20. What are the major exclusions in a medical insurance policy? Ans: The insurer shall not be liable to make any payment under this policy in respect of any expenses incurred by any insured in connection with Pre-existing illness or diseases Hospitalization during first 30 days of the cover War related injuries Elective cosmetic surgery Nonaccident related dental care Work related injury Fraudulent claims Eye test, glasses, hearing aid Experimental surgery 21. How is cumulative Bonus determined in a health insurance cover? Ans: The sum insured under the policy shall be progressively increased by 5 % in respect of each claim free year of insurance, subject to a maximum accumulation of 10 claim free years of insurance. In addition to cumulative Bonus, the insured will be entitled to reimbursement of the cost of medical check-up once at the end of every four underwriting years provided there are no claims reported during the block.

22. Discuss the implication of the following clauses in a health insurance policy PRE-EXISTING CONDITIONS CLAUSE POLICY REINSTATEMENT CLAUSE Ans: The object of the pre-existing clause is to prevent adverse selection. There may be some physical or mental conditions of the insured, which may have existed prior to the date on which the policy takes effect. Pre-existing conditions will be covered after a waiting period, which differs from insurer to insurer. Under the policy reinstatement clause, when the premium is not paid even within the grace period, the policy lapses. However the policy may be reinstated if the premium is received at least within 45 days. Sometimes a fresh application is required. There is a waiting period for sickness, not accidents, for the reinstated policy. PERSONAL AUTO INSURANCE 23. Discuss the nature and scope of the Personal Auto Insurance cover. Ans: A policy of motor vehicle insurance is, in ordinary course, a combined insurance. It insures the damage of the motor vehicle and its accessories, liability for damage for property, death of or injury to, the assured himself or spouse and it also insures the motor vehicle against the risk of liability for injury to, or the death of third parties caused by the drivers negligence. 24. Discuss the Property accident aspect of the motor insurance policy. Ans: In the motor vehicle insurance policy, if the motor vehicle is insured, the owner will be indemnified for any loss or damage caused to it by accident. Motor insurance being a contract of indemnity the insured is entitled to indemnity only, and that too in the manner stated in the policy. Medical expenses up to a limit are also payable. If the insured car is damaged, the insurer is entitled at his option to repair or replace the car or any part thereof or pay any amount of the loss or damage, in cash not exceeding the sum insured or the value at the time of loss whichever is less. If the part is not locally available or is exorbitantly costly to obtain from abroad, the insurer often limits the liability to paying in cash the catalogue price

issued by the manufacturer or his agent in India together with the cost of fitting such part. 25. What are the different types of policies available and what is the limit of indemnity under those policies? Ans: The terms of the policies define the nature and extent of the indemnity provided by the policy. There are two types of policies namely: i. The third party liability policy. ii. A comprehensive policy. The third party liability insurance is a compulsory under the Motor Vehicles Act. It is often said that a motor car policy is a unique combination of several types of General Insurance. For example a private motor car comprehensive policy indemnifies the assured against loss or damage to the insured car by accidental external means, by fire, self ignition, external explosion, lighting, frost, burglary, house-breaking or theft, and by malicious act. Thus it is very clear that the insurer is liable to make good the loss of a motor car to the owner of the car, for loss of car means loss to the owner of the car. 26. Describe the Personal Accident Aspect of the Personal Auto insurance policy. Ans: The personal accident aspect of the policy throws up on other risks, which an insured is likely to face. Besides, ensuring his personal safety under an ordinary policy, the extension clause indemnifies the assured for the injury caused to him whilst he is driving a motor car not belonging to him or hired to him and also any person driving the insured car on the insureds order or with his permission. Further, by paying extra premium, he may get extra cover over and above the general cover under the standard policy likea) Accidents to his wife and other specified relatives or friends; b) Loss or damage due to earthquake or flood, etc. The policy indemnifies the insured to the use of the insured car. However, it is extended in two ways, namely; it extends to the insured not only when he is driving his own insured car but also when he is driving a private car not belonging to him nor hired by him.

27. Mention the conditions in a motor vehicle policy to make the insurer liable. Ans: Some of the conditions in a motor vehicle policy to make the insurer liable are: a) The insured will maintain the vehicle in a good state of repair and efficient condition; b) He takes all reasonable steps and precautions to avoid accidents and to select competent and sober drivers ; c) He takes all reasonable steps to safeguard the car from loss or damage. 28. MENTION THE PROVISIONS AND CONDITIONS TO BE ADHERED TO, UNDER THE MOTOR VEHICLES ACT IN RESPECT TO THIRD PARTY INSURANCE. Ans: The Motor Vehicles Act has made it statutory and obligatory for a third party insurance cover to be taken by every owner of a vehicle. The Act specifically states that no person shall use except as a passenger or cause or allow any other person to use a motor vehicle in a public place, unless there is in force in relation to the use of the vehicle by that person or other person, as the case may be, a policy of insurance complying with the requirements of the Act. Thus, third party insurance is a must for running a motor vehicle in a public place. The following are some of the important provisions to be adhered to in case of third party insurance, namely i. ii. iii. iv. v. It applies to any other person other than a passenger; What is prohibited is the user by himself or allowing another person to use; Such use should be a motor vehicle; Such vehicles should be used in a public place; The using or causing of use by the other person should be without a policy of insurance. vi. The policy of insurance should comply with the provisions of the Act.

PERSONAL LIABILITY INSURANCE 29. WHAT ARE THE PERSONAL RISKS, A PERSON CAN INSURE AGAINST? Ans: A person can insure himself against the risk of death, personal injury or damage, deterioration or destruction of property, and also against the risk of incurring liability to third parties. 30. WHAT IS MEANT BY THIRD-PARTY LIABILITY? Ans: Third party liability is the liability which may arise by an insureds own conduct or in using his property, but still the risk of liability arising out of the use of the property is not covered by an insurance of that property. Liability policies are generally expressed as providing indemnity against liability in law. 31. WHAT IS THE MEANING OF LIABILITY IN /AT LAW? Ans: The phrase liability in/at Law is invariably understood and primarily used to cover the liability arising out of negligence. For example, the liability of a building contractor to a third party arising out of the faulty design of a structure was held covered though there was no negligence. Similarly, in a householders comprehensive insurance policy, the word accident covered nuisance liability which had occurred without the negligence on the part of the assured. 32. WHAT IS THE DIFFERENT INSURANCE COVERAGES OFFERED IN LIABILITY INSURANCE? Ans: Under the liability insurance category the following liability policies are covered: Public liability insurance, Liability arising in connection with professional negligence, Compulsory insurance, Employers liability insurance, Guarantee insurance.

33. EXPLAIN BRIEFLY EACH OF THE ABOVE MENTIONED LIABILITY INSURANCE POLICIES. Ans: Liability insurance policies are generally expressed as providing indemnity against liability in law. The various liability insurance policies are discussed at length below. Public liability insurance: Public liability does not mean liability of the state or its agencies. It means liability as imposed by law as opposed to self-imposed liability as in contract. The Public Liability Insurance Act, 1991, is intended to provide immediate relief to the persons affected by accidents occurring while handling any hazardous substance and for matters connected therewith and incidental thereto. In India this policy appears as a sequel to the famous Bhopal Gas Leak case. Professional Negligence: There are standard policies for professional indemnities cover for accountants, insurance agents, solicitors, and lawyers. These policies also cover the risk of loss by their own negligence. The law has made these professionals liable in respect of any loss or injury due to the negligence in the conduct of their professional duties. Compulsory Insurance: For the welfare of the employees, social welfare legislations have been passed in England and in India making it compulsory for employers to insure for safety of the workmen. The Indian Act makes it compulsory for the employer to insure his workmen by providing certain benefits to them in the event of their sickness, maternity and employment insurance. The employees insured under the Act are entitled to a) Sickness benefit b) Maternity benefit c) Disablement and Dependents benefit. Employers Liability: Though in olden days the liability of the employer has been extended in the law of torts by vicarious liability, in regard to his liability towards the employees, a number of defences were recognized, substantially reducing his liability towards his employee. For example, the doctrine of common employment, the defence of volenti non injuria were vital defences. But, in due course of time, the liability of the employer was extended due to the development of the industrial and labour welfare measures and legislations. Now the employers are tempted to take out insurances against such liabilities.

Guarantee Insurance: Guarantee business of insurance companies assumed great importance in the modern times. Earlier this was done by contracts of guarantee by which a friend or a relative of the promisor or employee used to stand as a surety for the due performance of the promise by the principal debtor or for the honesty of an employee. As the number of contracts increased, it became increasingly difficult to find sureties, as a result, chances of employment and business had to be lost. In such a situation, the insurance companies developed for themselves considerable amount of guarantee business. There are two methods by which this guarantee was given, namely; i. ii. the insurance company or the underwriter stands as a surety for the due completion of a contract or fidelity of an employee; and / or the underwriter insures the promisee or employer against the loss arising by non-performance of the obligor or the dishonesty of the employee.

The first types of contracts are simple guarantee contracts and only the second types involve element of insurance. The chief types of policies included in guarantee insurance are i. Insurance for performance of contract; ii. Insurance of debts; iii. Fidelity policies. A contract of guarantee insurance is a contract whereby the insurer undertakes to indemnify the insured from the loss caused as a result of the breach of contract or infidelity. Insurance for Completion of Contract: the subject matter of such contract is due performance of a contract. A enters into a contract with B but doubts whether B would complete the contract. In such a case A may insure Bs due performance of a contract. These are generally taken in cases of contracts of employment. Insurance for repayment of debt: A creditor may insure the repayment of debt, which he advanced or will advance in future. Such policies sometimes cover non-payments from specified causes only and in such cases only the causes for non-payment become relevant. When the creditor insures the repayment of a debt, on default by the debtor, the creditor can straight claim the money from the insurer. The insurance being a contract of indemnity, the insurer will be subrogated, on payment to the insured, to all the rights of the creditor against the debtor.

Fidelity Policies: These are the most common types of guarantee policies and are made usually for a term of one or more years. These arise generally out of the contract of employment where the employee has an opportunity to be dishonest. The risk covered is generally restricted to losses occurring while the employee is engaged in a specified capacity. Even in the employment, the risk covered may vary according to the specific terms of the policy in each case. For example, some are restricted to losses arising by embezzlement or fraud. Fidelity policies may be combined with liability policies, which are normally restricted to liability incurred through the negligence of the employee while the former policies are mainly intended to cover losses caused to the employer by the employees theft or embezzlement of money or securities. A fidelity insurer, like a fidelity guarantor, is entitled to subrogation. WORKMENS COMPENSATION INSURANCE 34. DISCUSS THE RATIONALE FOR THE EXISTENCE AND DEVELOPMENT OF WORKERS COMPENSATION CONCEPT AND PROGRAM. Ans: Workers time and again are injured and become sick because of job-related accidents and disease. Besides pain and suffering, these disabled workers also deal with other mental tensions and agony such as loss of earned income, payment of medical bills, partial or permanent loss of bodily functions or limbs or job separation.

Workers compensation is a social program that provides: Medical care Cash benefits Rehabilitation services to disabled workers from job related accidents or disease. These benefits reduce the economic insecurity that may result from job related disability. 35. STATE BRIEFLY THE OBJECTIVES OF WORKERS COMPENSATION CONCEPT. Ans: The following are the broad objectives of the workers compensation laws. Broad coverage of employees for job-related accidents and diseases. The workers compensation covers most occupation or job-related accidents and diseases.

Substantial protection against loss of income. The cash benefits are designed to restore a substantial proportion of the disabled workers lost earnings, so as to enable the disabled worker to maintain the same standard of living. Sufficient medical care and rehabilitation services. The workers compensation laws require employers to pay hospital, surgical, and other medical costs incurred by injured workers, as well as rehabilitation services to restore the disabled employees to productive employment. Encouragement of safety. The workers compensation laws encourage the firms to reduce job-related accidents and to develop effective safety programs. Experience rating is used to encourage firms to reduce job-related accidents and diseases, because firms with superior accident records pay lower WC premiums. Reduction in litigation. Workers Compensation laws are designed to reduce litigation by making prompt payment to disabled workers without requiring them to sue their employer. The objective is to reduce or eliminate payment of legal fees to attorneys and avoid delays in trials and appeals.

36. LIST OUT THE OCCUPATIONS NOT COVERED UNDER THE WORKERS COMPENSATION COVER. Ans: Although the WC laws cover most occupations, some occupations are either not covered or have incomplete coverage, namely: Farm workers Domestic servants Casual employees Concerns with numerical exclusions ( less than 3-5) Professional athletes REQUIREMENTS TO RECEIVE

37. WHAT ARE THE ELIGIBILITY WORKERS COMPENSATION?

Ans: There are two principal eligibility requirements to be met to receive the workers compensation benefits: The disabled worker should work in a covered occupation. The worker must have a job-related injury or disease.

This means the injury or disease must arise out of and in the course of employment. However, now the meaning of the term has been broadened so as to include the following: An employee who travels is injured while engaged in activities that benefit the employer. The employee is injured while performing specified duties at a specified location. The employee is on the premises and is injured while going to the work area. The employee has a heart attack while lifting some heavy material. 38. WHAT ARE THE BENEFITS PROVIDED UNDER THE WORKERS COMPENSATION INSURANCE? Ans: The Workers compensation insurance provides four principal benefits: Unlimited Medical Care: Generally medical care is covered in full. But medical care being expensive, and to hold down costs, some employers prefer tying up with some managed care organizations such as HMOs, and PPOs. Disability Income: The disability income is paid after the worker satisfies a waiting period that usually ranges from 3 to 7 days. If the injury lasts or the worker is disabled after certain number of days or weeks, benefit is given from retrospective date of injury. The weekly cash benefit is based on a percentage of the injured workers weekly wage typically 2/3rd , and is subject to minimum and maximum payments. Disability is classified into four categories: PT ( Permanent total) TT ( Temporary total) TP ( Temporary partial) PP ( Permanent partial) Temporary total disability claims are the most common and account for majority of cash claims. Death benefits: The death benefits are paid to the eligible survivors, if the worker dies as a result of a job-related accident or disease. Two types of death benefits are paid: Burial allowance

Weekly income benefits are paid to eligible survivors. Weekly incomes are a percentage of the diseased workers wages (2/3rd) and is paid to the spouse for life or until she/ or he remains, and also to a dependant child until a specified age. Rehabilitation services: In most cases, rehabilitation services are also provided to restore disabled workers to productive employment. Besides weekly benefits, boarding and room, travel, books and equipment charges are also given and also training expenses.

39. WHO IS AN EMPLOYER AS DEFINED UNDER THE WORKERS COMPENSATION ACT? Ans: An Employer is a person who employs workmen. The employer who engages a workman for casual duties (casual not by time) is not a workman within the meaning of the Workmens Compensation Act- 1923. Those engaged in the insureds business, profession, or occupation are workmen

40. WHY DO YOU ADVICE ALL EMPLOYERS TO TAKE THE POLICY? Ans: The liability of an employer is absolute by statute for the death of or bodily injuries or occupational diseases sustained by the workmen arising out of and in course of employment. Hence this policy is suggested to all the employers in their interest. FIRE INSURANCE 41. DEFINE FIRE INSURANCE. Ans: Fire Insurance is a contract of insurance by which the insurer agrees for consideration to indemnify the assured upto a certain extent and subject to certain terms and condition against loss or damage by fire which may happen to the property of the assured during a specified. There is said to be fire within the meaning of fire insurance when: a) There is actual ignition. b) The fire is purely accidental or fortuitous in origin so far as the insured is concerned. c) The fire has burnt/ damaged the property of the insured.

43. ENUMERATE THE ESSENTIAL CHARACTERISTIC FEATURES OF A FIRE INSURANCE CONTRACT. Ans : A contract of fire insurance is a species of a contract of insurance and it exhibits all the following characteristics, namely: a) This is a contract of indemnity b) It is a contract uberrima fides c) It must be distinguished from wagering contract and a contract of guarantee d) It is a personal contract and e) The cause of fire is immaterial generally. 44. STATE WHETHER A FIP IS A PERSONAL /IMPERSONAL CONTRACT? Ans: A contract of fire insurance though appears to be a property insurance is not so and it is a personal contract between the insurer and the assured, for the payment of money, in case the loss is occasioned to the property of the contracting party by fire. The purpose of the contract is not to insure the safety of the property but only to save the insured from the loss caused by damage to the property by fire. Therefore, where a property insured against fire is consumed by fire but still there is no pecuniary loss to the owner of the property, the insurer will not be liable to pay any amount. 45. IN AN FIP THE CAUSE OF FIRE IS SAID TO BE IMMATERIAL. JUSTIFY. Ans: In an FIP the cause of fire is immaterial. If the assured is careful and still there is fire, it would be unjust to disentitle him to claim and even when he or his servants are negligent and there is fire, even then it would be unfair to disentitle him to claim, for it is precisely for these reasons that a FIP is taken. One should understand that it is the damage and not the cause of fire that is insured. But in the following two case compensation is not givenWhen damage is caused voluntarily or wilfully When cause is within an exception of the contract.

TRAVEL ACCIDENT INSURANCE 46. DISCUSS THE SCOPE OF COVER AVAILABLE UNDER THE TRAVEL ACCIDENT INSURANCE COVER. Ans: The scope of the Travel Accident cover extends to all domestic travellers irrespective of the their age and income group. The benefits under this policy of insurance are very large and wide in depth for all those who frequently travel and face the various unforeseen risks of journey. 47. WHICH MODES OF TRANSPORT ARE COVERED UNDER THIS POLICY COVER? Ans: This cover is available to travellers by all modes: Road/ Rail/ Water/ Air, including by their own mode of transport. 48. WHAT IS THE BASIC OBJECTIVE OF THIS INSURANCE POLICY COVER? Ans: This cover is valid for transit period subject to maximum of 60 days. The cover includes incidental local travel also. The basic objective of this policy is to provide relief in case of accidental death and loss and /or damage to accompanied baggage during the travelling within the country. 49. WHO ARE THE PEOPLE COVERED UNDER THE POLICY COVER? CAN PA ALSO BE COVERED BESIDES THE AVAILABLE COVER? Ans: This policy is designed to provide cover for the insured, spouse, and dependant children only. Compensation under the Personal Accident will be in addition to all other existing covers or insurance covers insured might be holding. 50. WHAT ARE THE OTHER CAUSES OF LOSS AGAINST WHICH THE POLICY EXTEND ITS COVER? Ans: The policy covers reasonable and actual emergency incidental expenses upto Rs. 1000/- arising out of an accident resulting in a valid claim under a PA section. Besides, the loss or damage to accompanied baggage arising out of fire, storm, tempest, hurricane, flood, inundation, riot, strike, terrorism, malicious damage, accident, theft or burglary.

51. How is the limit of Indemnity fixed for this cover? Ans: The limit of indemnity under this cover depends up on the sum insured and otherwise as stipulated in the schedule. The sum insured per article is Rs. 500/- unless otherwise declared and claim of sum insured per article above Rs. 500/- should be settled on claimed value basis without applying depreciation and average. 52. WHAT IS THE UNDERWRITING FACTORS TO BE TAKEN INTO CONSIDERATION FOR FIXING THE PRICE? Ans: The pricing of the policy depends up on the number of persons as mentioned in the schedule. In case of more than 8persons, an additional premium of Rs. 40/- per head is charged. The service tax is charged extra. The cancellation of journey is to be intimated to the concerned authorities within 2 days of the schedule journey. Sometimes, extension of the cover will be granted subject to revised dates chosen by the insured, but not beyond a period of 60 days. 53. WHAT ARE THE GENERAL EXCLUSIONS UNDER THE TRAVEL ACCIDENT COVER? Ans: The general exclusions under the Travel Accident cover may be summarized as follows: Loss of articles of jewellry Jewellery made up of fully/ partially/ semi precious metal/ stones/ Money Securities Manuscripts Deeds Bonds Bills of Exchange, Promissory notes Stocks or share certificates Stamps and travel tickets or travellers cheques Business books and documents

Recoveries in baggage claims based on Subrogation principle may be waived if uneconomical.

ALL-RISKS COVER FOR JEWELLERY AND HIGHLY VALUED ITEMS 54. MENTION THE CATEGORY OF PEOPLE WHO CAN AVAIL THE ALLRISKS POLICY COVER. Ans: This policy is suitable for people owning Jewellery or valuables, which are prone to accidental loss or damage. 55. DISCUSS THE SALIENT FEATURES OF THIS POLICY. Ans: The policy covers valuables like Jewellery, ornaments, paintings, work of art, and similar artifacts of sentimental value, etc. The scope of cover is limited to loss or damage due to fire, riot, strike, terrorist act, burglary, housebreaking, larceny, theft, and accidental loss or damage. 56. WHAT ARE THE BENEFITS AVAILABLE UNDER THIS POLICY? Ans.: The policy pays for any loss or damage to the property insured caused by insured perils. The amount of claim payable would be limited to the sum insured or market value at the time of loss, whichever is lower. 57. MENTION THE EXCLUSIONS OF THIS COVER. Ans: The main exclusion of this cover is as follows: In case of paintings or work of art, other than damage by fire, partial losses are not covered. 58. WHAT ARE THE REQUIREMENTS UNDER THIS POLICY? Ans: Some of the important requirements of this policy are: Completed proposal form should give full and accurate information. All items to be covered should be fully described for easy identification in future. Unless each of the items is valued separately, the claim for each item will be limited to 5% of the total sum insured. Cover is not given freely on account of its vulnerability to loss and moral hazard.

SPORTS INSURANCE {GOLF INDEMNITY INSURANCE POLICY} 59. MENTION THE PEOPLE WHO ARE ELIGIBLE FOR THE COVER UNDER THE SPORTSMEN INSURANCE. Ans: Amateur sportsmen pursuing Cricket, Golf, Lawn tennis, Badminton, Squash, Angling, etc., and persons using sports guns can avail this policy. 60. WHAT IS THE SCOPE OF COVERAGE IN THE GOLF INDEMNITY POLICY? Ans: The policy is given to amateur sportsmen (not to professionals) for covering their sports equipment, personal effects, legal liability to third parties and personal accident risks. The cover can also be made available in respect of named members of insureds family residing with him. 61. WHAT ARE THE LOSSES COVERED UNDER THE SAID POLICY? Ans: The following losses are covered under the policy: Loss or damage to sports equipment, accessories, and wearing apparel to the extent of Rs. 2000 per person selected. The limit can be extended on payment of additional premium. Loss or damage to personal effects and wearing apparel while these are left in clubhouses, sports pavilion, etc. caused by fire, burglary, house breaking, or theft to the extent of Rs. 1000 per person for each sport selected. Legal liability of the insured and named members of the insureds family to the public while engaged in or practicing the sport, up to a limit of Rs. 500,000 for any one accident. Accidental bodily injury to the insured and to the named members of the insureds family while engaged in or practicing the sports up to Rs. 15,000. For Golf, the insurer will pay the insured at the rate of Rs. 200 in respect of each hole. 62. CAN SPORTSMEN STAYING ABROAD AVAIL THIS POLICY? Ans: No, the sportsmen staying abroad cannot take this policy. The geographical limit is usually India. But, however, the cover can be extended for events worldwide.

63. STATE THE UNIQUE FEATURES OF THIS POLICY. Ans: This is a unique policy designed especially for amateur sportsmen. The premiums depend upon the number of sports selected and the number of family members included. Some of the benefits offered by this policy are not covered by any other policy and therefore it is strongly recommended for amateur sportsmen. SECTION C CASE STUDIES 1. Mr. Dhanraj insured his new Santro car with Royal Sundaram Co. in the month of April 2002, and allowed his son to drive it. Dhanraj died in June 2002. His son continued to drive the car and caused an accident in the month of January 2003, and claimed indemnity under his fathers policy. Justify his claim. ANS: The insurance company contended that the permission lapsed automatically with the death of his father. Hence the son is not entitled for any compensation under the policy. Thus his claim is not sustainable. 2. Mr. Suresh, who was the owner of a car, sold his car for its price and when the cheque could not be encashed he claimed indemnity from the insurer for the loss of the car. Is his claim valid? Ans: In the above case, the claim put forth by Mr. Suresh is not justified because, as the court also held that the loss was the loss of the sale proceeds and not the loss of the car and so the insurer is not liable to pay any compensation. [Eisinger v GAFL]. 3. Mr. Ranjit Singh had engaged a driver for his car, who had negligently driven the vehicle and caused damage to a third party. Is Mr. Ranjit responsible in the above situation? Ans: As per the common Law, the master is liable for the tortious acts of the servant provided the servant does such act in the course of his employment. The common law also recognizes the vicarious liability of the owner of the motor car. In the law of torts, if a person negligently drives his vehicle and causes injury or death to the third party, the driver whose negligence caused the damage is liable to the third party. The driver is the servant of the owner, and since he is a person

of no means, the owner is liable for all his acts so far that he has done such acts in the course of his employment. In the above case based on the provisions of the common law Mr. Ranjit is held liable for the damage caused to the third party. [Pushpabai Sudershin v Ranjit G and P Co]. 4. Mr. Amar lost his factory furniture valued at Rs. 200,000 as on the date of fire. He wanted to recover a claim of Rs 2,50,000 for he had a policy of Rs 500,000. On enquiry he found that for new furniture the cost would be Rs 1,75,000. If the principle of indemnity is to be applied how much of his claim is to be accepted? Ans: Since the market value of the furniture lost was Rs 2,00,000 and the cost of reinstatement would cost the insurers only Rs 1,75,000, the Insurers would opt for reinstatement and accept the claim of Rs 1,75,000. 5.Mr. Kishore insured his machinery and stock of goods stored in the factory premises against damage by fire and a protection note was given, subject to the usual conditions of the companys policy, one warranty clause being smoking and cooking be strictly prohibited in or about the premises. The stocks were damaged by fire said to be of accidental nature. But the insurance company claimed that smoking a cigarette or bidi carelessly by some employee occasioned the fire. Is the denial justified? Ans: In the above case, the company denied the claim on the ground that there was a breach of warranty as the fire was occasioned by smoking which is strictly prohibited. But as there was no eye-witness to the origin of the fire, the court held that the cause of fire was a matter of conjecture. [Bhattacharjee v Sentinal InsuranceCo.] In the famous case Dekhari Tea Co v Assam Bengal Roadways Co, it was also held that fires cannot always be explained, and it must be a matter of conjecture. As regards the warranty, as the plaintiff had put notices strictly prohibiting smoking in and around the places, in fact there is no breach of warranty. Hence, the denial on the part of the insurance company is not justified. On the other hand, the company should make good the loss.

CHAPTER- 7 GENERAL INSURANCE (COMMERCIAL)


SECTION - A
1. Fire losses in a FIP is covered only when a.there is actual ignition b.the ignition is accidentally caused c.there must be a loss of some property d.all the above e.none of the above 2. Contractors all risk insurance forms part of a) Personal general insurance b) Commercial general insurance c) Group insurance d) Social security insurance 3. Professional indemnity insurance does not cover a) Acts of commission and omission b) Acts of negligence c) Liability arising out of failure of doctors treatment d) Criminal Act 4. The standard erection all risk insurance policies does not cover: a) Fire risk b) Negligence c) Storm caused damages d) Removal of debris

5. Implosion in fire insurance means b) Inward burst c) Sudden burst with loud report d) Damage caused by corrosion e) Damage caused by tempest 6. Forceful entry is the chief characteristic of a) Theft b) Arson c) Larceny d) Burglary SECTION A Answers : 1) d 2) a 3) d 4) d 5) a 6) d ANSWERS

SECTION - B
1. Enumerate the essential characteristic features of a Fire Insurance Contract. Ans : A contract of fire insurance is a species of a contract of insurance and it exhibits all the following characteristics, namely: f) This is a contract of indemnity g) It is a contract uberrima fides h) It must be distinguished from wagering contract and a contract of guarantee i) It is a personal contract and j) The cause of fire is immaterial generally. 2. What is the difference between subject- matter of insurance from the subject - matter of a contract of insurance? Ans: The main object of the contract of insurance is to indemnify the assured from the loss caused by damage or destruction by fire of the property of the assured. This physical object is called the subject matter of insurance. On the other hand the subject- matter of contract of insurance is not the physical object or property of the assured but is money and money alone. It must be noted that

what is insured is not the physical property of the assured but only loss of it by fire because by this contract loss by fire cannot be prevented. It is not an insurance against accidents but an agreement to protect against damage by a fire accident. 3. State whether a FIP is a personal /impersonal contract? Ans: A contract of fire insurance though appears to be a property insurance is not so and it is a personal contract between the insurer and the assured, for the payment of money, in case the loss is occasioned to the property of the contracting party by fire. The purpose of the contract is not to insure the safety of the property but only to save the insured from the loss caused by damage to the property by fire. Therefore, where a property insured against fire is consumed by fire but still there is no pecuniary loss to the owner of the property, the insurer will not be liable to pay any amount. 4. Can you have a separate consequential loss policy without an ordinary FIP? Give some examples of consequential losses. Ans: No, there cannot be a separate consequential loss policy without an ordinary fire policy. Under a FIP, not only can one insure the property but any consequential loss can also be covered. The claim under this head can succeed only if the insurers adjust their liability for loss of property by fire under an ordinary policy. The following are some of the examples of consequential loss for which indemnity is provideda) b) c) d) Loss of profits Standing charges Increased cost of working Increased cost of reinstatement

5. In an FIP the cause of Fire is said to be immaterial. Justify. Ans: In an FIP the cause of fire is immaterial. If the assured is careful and still there is fire, it would be unjust to disentitle him to claim and even when he or his servants are negligent and there is fire, even then it would be unfair to disentitle him to claim, for it is precisely for these reasons that a FIP is taken. One should understand that it is the damage and not the cause of fire that is insured. But in the following two case compensation is not givenWhen damage is caused voluntarily or wilfully When cause is within an exception of the contract.

6. State the significance of the doctrine of Proximate Cause with respect to fire insurance claims. Ans: The doctrine of Proximate Cause holds a very significant place in the determination of fire related claims. Proximate cause means the active efficient cause that sets in motion a chains of events which brings about a result without intervention of any force and working actively from a new and independent source e.g. where insured object is burnt, cause is plainly fire and insured is entitled to recover unless the insurer can show that the fire was caused by exempted peril or willfully by insured or with his consent such as: 1. Where there is an explosion during a fire, the concussion damage falls within exception and the Insured cannot recover. 2. Where subject matter is burnt but fire, which burned it, was due to natural consequences of excepted peril- Insured cannot recover. 3. Where fire was set in operation by earthquake, if not covered by insured under fire policy and then spread by natural causes i.e. spread by wind or one thing catching fire from another and so on. Insured cannot recover. 7. What is the scope of coverage under the Industrial All Risk Insurance Ans:Industrial all risk insurance is available to all major industrial units (other than petrochemical risks) both manufacturing as well as storage, having all sum insured of Rs.100 crores and above, in one or more locations in India. The policy covers damages that are covered under: i. ii. iii. iv. Fire and special perils policy. Burglary insurance policy. Machinery breakdown/ boiler explosion/ electronic equipment policy. Business interruption due to fire and special perils policy.

The policy excludes the following: i. Inherent vice, defects, deteriorations and normal wear and tear. ii. Faulty material/ workmanship/ defective design and material. iii. Pollution, contamination, shrinkage, rust, corrosion, scratching and temperature changes. iv. Collapse or cracking of buildings. v. Willful act, negligence, war and nuclear risks.

vi.

Larceny, fraud, dishonesty, inventory losses, shortage on delivery.

8. Discuss the comprehensive cover under the CAR policy Ans:The CAR policy provides the following covers for losses due to: (a) (b) Collapse, collusion, impact. All the accidental events during construction like dropping or falling, defective workmanship and material, lack of skill, negligence, malicious act and human error (handling risks). Lack of skill, negligence, malicious and terrorist damage (risks of human element). Failure of safety devices, leakage of electricity, failure of insulation, short circuit, explosion, etc. (operational risks). Natural calamities like flood and earthquake, inundation, etc.

(c)

(d) Theft and burglary, fire / lightning (location risks). (e) (f)

9. How is the premium for a CAR policy determined? Ans: The factors that effect the risk associated with the project are considered while working out the premium. They include: 1. 2. 3. 4. 5. The nature of the project The project cost The project period Geographic location The period of testing

If the sum insured in the project exceeds Rs.100 crores with the option of higher deductibles, then a discount in premium is allowed. The discount is also given if the project site is protected against the fire. Projects with period of more than one year are allowed to pay the premium in instalments. Where projects get completed before the estimated time period, the premium for the balance period of insurance may be refunded. 10. How is the EAR policy different from the CAR policy? Ans:Erection all risk insurance is similar in its nature to the contractors all risk insurance. It provides cover for the erection of the machinery. It is a comprehensive insurance policy that covers all the risks right from the beginning when the materials are unloaded at the project site till the time project is tested,

commissioned and handed over. The erection all risk policy is appropriate for projects involving standing structures. Such structures are exposed to various risks during the construction period like the damage of the plant and the machinery or the supporting structures. 11.How is Machinery Insurance policy different from profits policy? Machinery Loss of

Ans:Machinery (breakdown) insurance policy covers all the costs involved in bringing the machinery back to pre-breakdown state. Machinery Insurance Policy covers almost all the stationary capital equipment. The policy covers all sorts of events that can cause breakdown of the machinery. All accidental, electrical, mechanical breakdowns due to internal causes, external causes, operational deficiencies or human errors are covered under the policy. The Machinery Loss of Profits Insurance primarily aims at covering the consequential losses, say, due to simultaneous failure of standby equipment. Machinery loss of profits insurance helps to make good such losses. It minimises the effects of the consequential losses during the interruption period and helps the entrepreneur to restore the business operations to normalcy. The policy covers the losses due to 1. Decrease in turnover 2. Rise in cost of working The losses as a result of reduced turnover because of the damaged machinery and the additional expenditure necessarily incurred for avoiding or reducing the fall in turnover for the interruption period is compensated under this policy.

12. Does a Boiler explosion policy cover loss due to fire? Ans:This policy covers loss or damage to steam generating equipments like boilers and other fired and unfired pressure vessels against the risk of explosion and collapse due to internal pressures that are inherent in all such equipments. The policy covers the damage caused to the boilers and other pressure plant due to explosion (not due to fire). For additional premium, damaging to the surrounding property of the insured, liability on account of damage to the property not belonging to the insured, can also be insured. So a businessperson should avail this policy to be fully secured against the losses due to fire and explosions.

13. How is a burglary policy beneficial for a commercial business unit? Ans:The policy covers the following: Stock in trade Goods held in trust or on commission Fixtures and fittings, plant and machinery that is movable property

The policy offers protection against burglary and housebreaking. Forceful entry is the chief characteristic of burglary. Other crime perils like theft, larceny, robbery etc. are not covered under this policy. The policy requires proper care of the insured property. If the cash in the safe is insured, the safe should be locked and keys of the lock should not lie anywhere near the safe 14. How and when does liability arise under a product liability Ans:Liability arises out of 1. 2. 3. Tort or common law Statutory law quite often it is absolute or no fault liability Contract

Product liability insurance covers the damages arising in such cases. And the damages arising or the compensation that is required to be paid in such case is huge. Hence all the industries that are exposed to such risks should go for product liability insurance cover, even if it is not mandatory for them to avail this cover. 15. What is the scope of cover available under a professional liability policy? Ans:Professional indemnity insurance covers the professionals against all the liabilities that may arise due to the negligence or failure in providing the service. That is why this policy is also called Errors and Omissions Insurance (E&O Insurance) or malpractice insurance. 16. Who are defined as professionals? Ans:Initially the policy was designed for professionals like doctors, lawyers, architects or engineers but now the policy is widening its scope to accommodate emerging professions. The policy now includes various professions like

psychiatrists, marketing or technology consultants, environmental consultants, insurance agents, brokers etc.

software

designers,

17. How is the professional liability cover different from other liability covers? Ans:The professional liability insurance differs from other liability insurance policies in a few ways. These are as follows: 1) While other liability insurance policies usually specify the per occurrence limit, there is usually a maximum limit for each claim, but there is no limit per occurrence in case of a professional liability policy. Further no distinction is made between bodily injury and property damage liability. Professional liability insurance is not restricted to accidental acts: faulty diagnosis or faulty performance is also covered. Deliberate acts giving unintended results are also covered in the policy.. Professional liability policies usually cover the damage caused to the property in the custody or care of the insured as well.

2)

2)

4. Professional liability insurance does not allow the settlement of the claim without the prior approval of the insured. 18.Is a doctor also equally liable for the negligence of his assistant and nurse? Ans:The doctor professional liability policy protects the doctor for the act of the following people such as: The acts of the qualified assistants and the employees of the insured who are named in the policy are also covered in the policy The acts of the qualified assistants and the employees of the insured who are named in the policy are also covered in the policy The claims should relate to the acts or omissions committed during the period of the policy The limit of the indemnity granted under the policy for Any One Accident (AOA) Any One Year (AOY) (per accident per policy year) will be identical

19. What is a respondentia bond? Ans:Respondentia loans were given to the captain of the ship against the cargo to ensure the safety of journey. They were paid back only if the cargo reached the destination safely.

20. What are bottomry loans? Ans:Bottomry loans in contrast to Respondentia loans were raised against the ship/vessel. High premium was charged on the loan to cover the risk. Hence in many respects they were similar to present days marine cargo and hull insurance. 21. Define the contract of marine insurance? Ans:Marine insurance is a contract between the insurer and the insured. The insurer as per the agreement undertakes to indemnify the insured in an agreed manner and appropriate against marine losses incident to marine adventure. The insured in return pays the premium to the insurer. 22. Define the scope of coverage under the MIP ? Ans:The coverage under marine insurance policy includes the following: The ship (hull and machinery) The insurable goods and property exposed to maritime perils. Other incidental earnings like freight, commission etc. which will be lost along with the property due to the maritime perils. The third party liabilities incurred by the insurer or other person responsible for or interested in the property. Expenses incurred to prevent and minimise losses

Maritime perils are the perils that are incidental to the sea journey that arise in consequence of the sea journey. 23. Why is a marine policy called as valued policy? Ans: Marine insurance is a valued policy because the value is agreed upon in advance, unlike fire and other insurance where the indemnity is limited to the actual loss. This value might be more or less than the actual loss occurred. 24.. What is the protection assured under a crime insurance policy? Ans: There are two types of financial protection that are available against the losses caused by crime. They are fidelity and surety bonds and burglary, robbery and theft insurance. A bond is a legal instrument in which a third person (surety) ensures the performance of contract properly by the principal or the obligator. A

Fidelity bond deals with assurance of bonafide behaviour by an employee during the course of his employment. In fidelity bond, the surety assures the employer of trustworthiness and honesty of the employee and agrees to pay the damages that arise due to the dishonest acts of that employee. 25. Differentiate between a fidelity bond and crime insurance. Ans:While in a bond contract the surety is required to pay for any losses, he also reserves the right to recover it from the defaulting principal. Whereas in case of an insurance contract, the insurer is prepared to pay for the loss and works on the principal of spreading this loss over the group of insured people. Secondly the matter covered by bonds is under the control of the insured and the losses covered by insurance are matters outside the control of the individual. Thirdly while the insurance contract is cancelable, usually, by either of the party, the bonds cannot be cancelled until all the obligations of the principal are fulfilled. Lastly, insurance contract involves two parties, whereas bonds involve three. 26. Discuss the scope of coverage under ICC clauses for marine policy? Ans: Institute Cargo Clauses C ( Basic Cover) covers the following : a. i. Loss or damage to the subject- matter insured reasonably attributable to: Fire or explosion Vessel or craft being stranded, grounded, sunk or capsized Overturning or derailment of land conveyance Collision or contact of vessel / craft or conveyance with any external object other than water Discharge of cargo at the port of distress

ii. Loss / damage to the subject matter insured caused by: General Average sacrifice Jettison

b. General Average contribution and Salvage charges incurred to avoid loss from any cause (s) except those excluded c. Liability under Both to Blame Collision clause of contract of Affreightment

a.

Institute Cargo Clause B covers the aforesaid risks of ICC (C) and i. Loss / damage reasonably attributable to earthquake, volcanic eruption or lightning; ii. Loss or damage caused by - Entry of sea, lake, or river water into vessel, craft, lift van or place of storage - total loss of any package lost overboard or dropped whilst loading into or unloading from vessel or craft General Average contribution and Salvage charges incurred to avoid loss from any cause (s) except those excluded Liability under Both to Blame Collision clause of contract of Affreightment Institute Cargo Clause A Comprehensive cover

b. c.

a. All Risks of loss / damage to the cargo insured except those specifically excluded. b. General Average contribution and Salvage charges incurred to avoid loss from any cause (s) except those excluded c. Liability under Both to Blame Collision clause of contract of Affreightment 27. Enumerate the general exclusions under the marine insurance policy? Ans: The exclusions under a marine insurannce policy include the following: - Loss, damage or expense attributable to willful misconduct of the insured - Ordinary leakage, ordinary loss in weight or volume or ordinary , wear and tear - Inherent vice or nature of the subject matter - Delay - Insolvency or financial default of the owners, operators, etc of the vessel - Insufficiency or unsuitability of packing - War and allied perils - SRCC- Strike, Riot, Civil Commotion and Terrorism

28. What are the extension of Cover Ans:The ICC B cover for extraneous perils include; a. b. c. d. e. f. g. h. i. j. Theft, pilferage, and non-delivery Fresh / rain water damage Damage by hook, oil, mud, or acid Breakage or leakage (not ordinary leakage) Country damage Shortage Bursting and tearing of bags SRCC Risks under all the clauses can be extended @ 0.0225% Ocean transit policies can be extended to cover War & SRCC @ 0.02475% Air transit policies can also be extended to cover the war and SRCC @ 0.025% additional premium.

SECTION C CASE STUDIES


1. Govind Rao, a dealer in timber, supplied timber to companies on a contract basis. He insured his business with Unity Insurance Company. Govind Rao had different payment arrangements with each of his clients. While some clients were asked to pay in cash, others were allowed credit. Green Earth Public Ltd was one of the companies that purchased timber from Govind Rao. Govind Rao received fully paid shares of the company as payment for the timber sold to it. By financing the company during its time of need, Govind Raohad also become an unsecured creditor of the company. On 10April 2000, Govind Rao and Green Earth signed a deal for 1000 teak wood logs. The delivery of the timber was to take place the next day. Unfortunately, that night there was a short circuit at the godown where the timber was stored, and the entire consignment was destroyed in the fire. Govind Rao filed a claim with the insurance company for the loss he had incurred. Unity, however, rejected his claim and refused to pay for the loss, alleging that the timber which was destroyed was sold by the insured to Green Earth and, therefore, Govind Rao no longer had any claim over it.

Questions for discussion: 1. Is Govind Rao entitled to receive compensation from the insurance company for the loss caused by the fire? 2. What is insurable interest? What are the policy requirements for an insurance interest? Ans: 1. After having finalized the sale of the timber to Green Earth Public Ltd., Govind Rao has no insurable interest in the timber since it has become the property of the buyer, that is, Green Earth. Insurable interest is the legal right of the owner of a property to insure the property. One of the conditions of an insurance policy is that the policyholder should have an insurable interest throughout the period of the policy. Further, during the tenure of the policy, the policyholder may lose his insurable interest by way of transfer of ownership, etc. In such a situation, he is no longer entitled to claim for loss to the insured property as the sold property no longer belongs to him. Moreover, a shareholder or creditor of a company cannot insure the assets of a company, since the assets belong to the company. The company, which is an artificial person, has an insurable interest in the companys assets. Thus, in the given case, the timber has become the property of Green Earth and Govind Rao is not entitled to receive any compensation from the insurance company for the loss to the timber due to the fire. 2.The owner of a property has the legal right to insure his property if its loss or damage is likely to affect him financially. This legal right of the owner is called insurable interest. The absence of an insurable interest renders an insurance policy void because one of the conditions of the policy is that the policyholder maintain an insurable interest throughout the period of the policy. Instead of making the existence of an insurable interest a precondition for obtaining coverage, insurance policies, instead, limit the payment on any claim to the extent of the insureds interest. In addition to this, proof of loss forms that accompany insurance policies require the insured to specify all interests that he may have in the property. Limiting the payment of claims to the extent of an insureds interest and requiring the insured to specify all interests in the property is essential for claims adjusting. If it were possible for an insured to be able to collect more than the interest he has in the insured property, it would serve as an incentive

for the insured to cause deliberate destruction to his property. Also, an insureds interest can change due to circumstances such as marriage, divorce, or additional mortgages. This can create opportunity for false claims if the amount recovered by the insured as insurance is not limited to the insureds actual interest. By identifying all interests in the property, an adjuster is able to treat all parties fairly, without compromising on the insurers rights. It also enables the adjuster to identify other coverages. 2. Shankar Raman joined the railways in 1975. He started out as a mechanic and worked in different capacities from 1975 to 1998. As a mechanic, he had to use solvents to remove engine grease from engine parts and tools as well as from his hands and clothing. After some years he got promoted as supervisor. However, his exposure to solvents did not stop there. As a supervisor, he continued to inhale solvents during the daily inspections. After some time, Raman experienced rashes, dizziness, breathlessness, and disturbed sleep patterns. Because of his problems, he even made life difficult for other members in his family. Raman usually maintained a good rapport with his peers and superiors. But in 1990, he had problems with his immediate superior Pawan Sarkar. Sarkar even allegedly waged a campaign of harassment and intimidation against Raman. Despite his problems, Raman never complained against Sarkar to higher authorities. Unable to bear the harassment, Raman thought of quitting the job. But his obligations tied him down. So he faced all kinds of problems without making an issue of it. In 1995, Raman began to suffer from weight loss, headaches, nausea, anxiety, memory loss, and other health problems. He even sought psychiatric treatment for depression in 1995. One of the psychiatrists attributed Ramans mental problems to job stress. In 1998, unable to continue with the organization any more, Raman decided to quit the job.

Even after one year of quitting the job, Raman could not recover completely. He approached Ayurvedic doctors and used herbal medicines. Unfortunately all his attempts were rendered futile. Raman then claimed compensation from the railways for the ailments he had contracted during the tenure of his service and the financial burden he had to bear in the process of undergoing treatment for these ailments. The railways forwarded Ramans claim to its insurer Fair Insurance Company. The insurer, in turn, appointed a claims adjuster to carry out the necessary investigations and to decide on the claims amount it had to pay Raman. The claim adjuster surveyed the workshop premises where Raman used to work, spoke to Ramans colleagues and subordinates, and noted down a few things to discuss later with the management. The claims adjuster even obtained the addresses of the doctors who had treated Raman and had detailed discussions with them about Ramans health problems. Later, while giving the management his report on what the claim amount should be, the claims adjuster stressed on some vital aspects of Ramans job which, if suitably modified would not only help Raman come back to work without causing any further harm to his health, but would also help the railways lower the disability expenses they were required to pay Raman. Questions for Discussion: 1. Claims adjusters handling workers compensation cases must investigate them diligently. Discuss, how well the adjuster investigated Ramans compensation case. How will the compensation adjusters report to the railways management help control disability expenses in future?

2.

Ans: 1. There is no real investigation into workers compensation cases that only involve medical expenses. The claims adjuster takes the policyholders word as proof that the accident happened on the job and that the injury is work-related. The adjuster usually conducts an investigation if the accident involves lost time from work. In such cases, he obtains statements from the claimant, the employer, and any witnesses present. With the help of these statements, the adjuster tries to establish whether the injury is workrelated or not, if the relationship between the employee and the employer

might have prompted exaggerated claims by the claimant, and whether the injury was preexistent or not. The adjuster must also obtain documentation of the earnings of the employee in order to calculate the employees disability compensation accurately. 2. For compensation adjusters, controlling disability expenses is probably the most important issue. Cases in which the claimant returns to work promptly and does not lose any time from work are relatively simple and straightforward for the adjuster to handle. The biggest problem that compensation insurers face are from cases which involve disability extending over a long period of time. Compensation insurers cannot simply stop paying compensation because they believe that the disability should have ended. Once the insurer accepts a case as being compensable, he cannot end disability payments except by an agreement with the claimant to do so or through an order of the compensation commission. If the claimant disagrees with the insurer, it may take months to resolve the case before the compensation commission. The decision of the compensation commission is generally in favor of the claimant. The insurer can stop payment if the commission allows stoppage of payment to the claimant and rules in favor of the insurer. However, in such cases, the claimant need not reimburse past payments. Adjusters can control disability expenses by insisting that the treating doctor give reasons for why the claimant cannot perform his job. This is necessary because many physicians certify disability without understanding the physical demands of the claimants work. For almost any kind of impairment, there are jobs, or certain aspects of a job, that can be performed by the affected person. Therefore, the physician cannot assume a disability purely on the basis of an impairment. Adjusters and employers can work together to modify the job of an employee by removing the most physically demanding parts from the job. On their road to recovery, claimants can be given work that have limited duties. The claims adjuster can also control disability expenses by constantly encouraging claimants to return to the job. In the process, they can find out from them the aspects of their work which they are still unable to perform and suggest the required modifications in the job to the employer, which can help the claimant to come back to work quickly.

CHAPTER 8 LIFE INSURANCE SECTION -A


1. Which of the following needs are satisfied by life insurance products? a. Risk coverage. b. Compulsory savings. c. Tax saving d. Both (a) and (b) above. e. All of (a), (b) and (c ) above. 2. What is the exemption under the condition of automatic renewal available to the insured? a. The insured need not have to pay the premium. b. The insured need not have to put his consent again. c. The insured need not have to undergo any medical test. d. The insured need not have to pay any extra premium for the higher attained age. e. All of the above. 3. What is the basis on which the premium should be charged in case of renewal of the term insurance policy? a. Attained age of the insured. b. Age of the insured at the time of taking the policy. c. Average age of the insured at the time of commencement of the policy and the maturity of the policy. d. Either (a) or (b) above. e. Any one of (a), (b) or (c ) may be chosen 4. Till when is the insured liable to pay the premiums in case of a limited payment whole life insurance policy? a. The insured is required to pay the premiums throughout the old age. b. The insured is liable to pay the premiums for a limited period. c. The risk of death coverage is available for a limited period only. d. The liability of the life insurer will gradually decrease to a limited value. e. The liability of the insured to pay the premiums will gradually increase but it will be limited to a certain value.

5. Which of the following policies by nature constitute thrifty savings? a. Term insurance b. Pure endowment. c. Endowment assurance. d. Term endowment. e. Pure insurance 6. Which policy do you suggest to manage the risk of excessive longevity? a. Term insurance b. Pure endowment. c. Endowment assurance. d. Term endowment e. Pure insurance. 7. Which of the following life insurance policies do not assure the minimum return for the investment component? a. Universal life insurance policy b. Variable life insurance policy. c. Endowment assurance policy d. Both (a) and (c) above e. Both (b) and (c) above 8. What are the advantages of the universal and variable life insurance policies? a. Income tax-deferred cash accumulation. b. Guaranteed death benefit and other optional benefits. c. Flexibility about the amount of premium payments. d. Choice of the timing of premium payment to the policyholder. e. All of the above 9. What components does very payment in case of an annuity certain consist of? a. Principal component b. Interest component c. Risk premium component. d. Both (a) and (b) above e. All of (a), (b) and (c ) above.

10. Which of the following statement(s) captures the similarities between a life insurance and an annuity contract? a. Both protect against the absence of any incomes. b. Both the contracts together complete the economic program of a person. c. Rate of mortality influences both types of coverages d. Moral hazard may arise in both the situations. e. All of the above 11. What is the initial payment by the annuitant known as in an annuity contract? a. Purchase price b. Premium c. Cash value. d. Policy value. e. None of the above. 12. Which of the following statement(s) is/are true regarding an annuity plan? a. Annuity payment must be started immediately on receipt of the purchase price in all annuities b. Annuity plans are generally undertaken to cover the risk of death c. A single annuity plan can be undertaken by more than one annuitant. d. Insurer is liable to refund the purchase price to the nominee of annuitant in every case. e. All of the above. 13. Which of the following statement(s) is /are true regarding the amount of annuity installment payable by the insurer? a. b. c. d. e. It is not related to the entry age of the insured. It increases with the entry age of the insured. It decreases with the entry age of the insured. It cannot be predicted so easily. It depends on many other factors along with age.

14. For which annuity, the investment risk is an integral part? a. Fixed annuity. b. Variable annuity. c. Immediate annuity. d. Deferred annuity. e. Annuity certain. 15. What does deferred annuity plan consist of? a. Accumulation period only. b. Liquidation period only. c. Payment period only. d. Both (a) and (b) above. e. All of (a), (b) and (c ) above.

SECTION - A ANSWERS
1. (e) Life insurance products offer good savings plans along with different risk coverages that are related to our life. Life insurance savings also offer some tax incentives to the policyholder as per the income tax rules. 2. (c ) Under the conditions of the automatic renewal feature, the insured can renew the insurance policy at the end of each fixed period without undergoing any medical test in order to prove his/her insurability. 3. (d) An insurer may renew the policy on the basis of any one of the given two ages of the policyholder. If the policy is renewed on the basis of the attained age of the insured, higher amount of premium due to the increased age is to be paid by the insured. If the age at the commencement of the policy is chosen for renewal, the difference in premium amount is to be paid along with interest. 4. (b) Under a limited payment whole life policy, the insured is liable to pay the premiums up to a certain period of time. But in return of those premiums, the risk of death coverage is available at any time irrespective of the age. 5. (c ) One can build up certain amount of corpus based on the future needs at any expected point of time in future. Due to the very nature of the contract, one is forced to save and the proceeds of policy are payable only on maturity or on early death which may be predetermined on the basis of the amount and time of the expected funds requirements.

6. (b) In case of pure endowment policy, the amount of sum assured is paid to the policyholder on survival till the end of the policy term. During old age, funds are required to meet the minimum physiological needs while the working life of a person comes to an end. Therefore, such a policy can offer the best choice to manage the risk of the excessive longevity at a low cost. 7. (b) Under the variable life insurance policy, the policyholder is required to choose the nature of instruments where the insurer can make the investments. By following that norm, the total proceeds are returned to the policyholder on maturity, without any assurance of even the minimum return. 8. (e) All the given features are applicable to both the policies 9. (d) In case of an annuity certain, the insurer pays a fixed sum of money to the annuitant for a certain number of years as agreed upon by both the parties. This fixed amount consists of principal and interest. In the initial years, the percentage of interest is more while it is the reverse in later years. 10. (e) In both the cases of annuity and life insurance, the insurer protects against the loss of incomes to the insured based on the mortality rate experienced by the insurable population. Life insurance contracts assure the risk of premature death while annuities are meant for the problems of excessive longevity. But in both the cases, the insured may distort the figure relating to the actual age that may be taken as a moral hazard. 11. (a) The annuitant is required to buy the suitable annuity contract by paying the proper price of the same. The price may be either in lump sum or in installments as per the capacity of the annuitant. The price paid by the annuitant is known as the purchase price. 12. (c ) A single annuity plan may be undertaken by a person individually or jointly with others depending on the needs of the annuitant. For example, one may take an annuity plan for self and spouse to get the assurance of regular cash flow for self as well as spouse till death. 13. (b) In annuity contracts, an insurer assumes the risk of survival of an annuitant. As the age of the person increases, the possibility of survival decreases. With increasing age, the risk gradually decreases and hence the amount of installments payable increases.

14. (b) The annuitant is required to pay the purchase price first to enable the insurer to start the installment payments as per the terms of the annuity. The insurer invests the purchase price to suitable financial instruments that yield the required patterns of cash flows. In variable annuity, the cash flows depend on the performance of the portfolio where the proceeds are invested. 15. (d) During the accumulation period, the purchase price is invested in suitable financial instruments to reach a certain value along with interest. Then this value is gradually paid to the annuitant regularly as per the terms of the contract. The first period is called as accumulation period while the second one is the liquidation period.

SECTION B
1. Describe the basic characteristics of term life insurance? Ans. Term insurance has several basic characteristics. Protection for a limited number of years.[temporary protection] Unless the policy is renewed, the protection expires at the end of the period. These policies have no cash value or saving element. Nothing is paid in case of survival. Issued for one year to a set number of years such as 10 or 20 or to a stipulated age 65 or 70. Initial premiums are low. Premiums for term coverage can escalate rapidly as the duration of the policy lengthens. These prices are more easily compared than are prices of other products. Hence this market is more price competitive than the market for cash value policies. These policies have no cash values, no dividends - so easy to compare on the basis of premiums. Term lapse rates are higher than other policies [ because of sensitivity in price and low profit margin.] Most term insurance policies are renewable, for additional periods without evidence of insurability with increased premium at each renewal based on insured' s attained age. Most term insurance policies are convertible for a cash value policy without evidence of insurability. This conversion may be permitted on an attained age or original age basis. Traditional term premiums often are based on aggregate mortality experience.

Reentry term premiums are based on a select \ ultimate mortality split. Hence the scale of premium rates varies not only by age but also by the duration since the insured last demonstrated insurability. 2. When is the use of term insurance appropriate? Ans. Term insurance is appropriate when income is limited, or there are temporary high insurance needs. It cannot be used for retirement or saving purposes because it has no cash values. 3. what are the limitations of term insurance plans? Ans: Term insurance premiums increase with age and eventually reach prohibitive levels. Thus term insurance is not suitable for individuals who need large amounts of life insurance beyond age 65 or 70. Term insurance is inappropriate if we wish to save money for a specific need like child' s education, or for retirement because these policies do not accumulate cash values. 4. Describe the whole life insurance. Ans. Whole life insurance is a cash value policy that provides lifetime protection. THE NATURE OF WHOLE LIFE INSURANCE: It provides for the payment of the face amount upon the insured' s death regardless of when death occurs The face amounts payable under whole life policies typically remain at the same level through out the policy duration, although dividends are often used to increase the total amount paid on death In most policies the gross premium also remains at the same level through out the premium payment period with some exceptions. Universal life policies can function as whole life insurance if they have sufficient cash value WHOLE LIFE CASH VALUES: All whole life policies involve some pre-funding of future mortality costs Cash values are available to the policy owner at any time by surrendering the policy Facility of obtaining a loan with rate of interest upto that of the policy' s cash value Loan is deducted from the gross cash value when death claim is payable Whole life policies may be participating or non participating with some non guaranteed element.

5. Describe the limited payment whole life insurance Ans. LIMITED PAYMENT WHOLE LIFE INSURANCE policy remains in full force for the whole of life but premiums are payable for a limited number of years only, after which the policy becomes paid up for its full face amount. It is different from vanish pay policy where there is no guarantee of not required payments beyond the stated premium payment period . The greater the number of premium payments the more closely the contract resembles the ordinary life form. Annual level premiums of these policies are larger than ordinary life plan because the actuarial equivalent of the premiums payable for the insured' s entire lifetime. The higher the premiums for a policy, the greater the cash values . The size of the cash value varies inversely with the length of premium paying period. It contains the same non-forfeiture, dividend, settlement options of ordinary life policy. The one end extreme in limited payment life insurance is the single premium whole life policy and another end is ordinary life insurance. It is not well adopted to those whose income is small and whose need for insurance protection is great. This policy fit many business insurance situations 6. What is an endowment insurance? Ans. An endowment policy pays the face amount of insurance if the insured dies within a specified period; if the insured survives to the end of the endowment period, the face amount is paid to the policy owner at that time. The mathematical concept of endowment insurance = term life insurance + pure endowment [ to pay the face amount if the insured dies during the period + to pay the maturity amount only if the insured is living at the end of a specific period, with nothing paid in case of prior death.] The economic concept divides endowment insurance into two parts : decreasing term insurance and increasing savings.

7. Explain the major characteristics of universal life insurance Ans. Universal life insurance is an important variation of whole life insurance. It can be defined as a flexible premium policy that provides lifetime protection under a contract that unbundles the protection and saving components. It is sold as an investment rather than as protection. Major characteristics: Greater premium flexibility [ frequency in mode of payment and in the amount of payment subject to sufficient cash value to cover mortality costs and expenses.] Adjustable face amount Contemporary interest rates Unbundling of the savings, expenses, and protection elements and associated pricing. Increased disclosure through the statement which shows the mortality charge for the cost of insurance, expense charge for sales and administrative expenses, and interest credited to the cash - value account. [annual premium expense and administrative charges - mortality charge + interest = cash value account at end of year.] Two forms of universal life insurance 1.pays a level death benefit during the early policy years 2. An increasing death benefit.[ This is more expensive because the insurer must pay a higher death benefit.] A partial cash withdrawal [ not a loan] can be made without terminating the policy. Interest is not charged for this, but the death benefit is reduced by the amount of withdrawal. Policy loans are permitted at competitive interest rates. If the policy permits, additional insureds can be added to the policy. Favorable income tax treatment as traditional cash value policies. Limitations: Misleading rates of return [insurers advertise gross returns which does not reflect sales commissions, expenses and the cost of insurance protection.] Incomplete disclosure [ the policy owner is not given information on how the expenses are allocated between the protection and saving components in the policy] Decline in interest rates [ when there is a decline in interest rate the cash value and premium payment projections based on higher interest rates are misleading and invalid ] Administrative costs are high. Uncertainty cash flows has proven a challenge operation of this policy. Insurer has the right to increase current mortality charge up to the maximum limit

Policy owner often lacks a firm commitment to pay premiums. As a result the policy may lapse because of non-payment of premiums. 8. Define an annuity Ans. An annuity can be defined as a periodic payment that continues for a fixed period or for the duration of a designated life or lives. The person who receives the periodic payments or whose life governs the duration of payment is known as the annuitant. The fundamental purpose of a life annuity is to provide lifetime income that cannot be outlived. 9. Classify annuities. Ans. Annuities may be classified in numerous ways, based on number of lives covered method of premium payment time when income begins method of disposing of proceeds Denomination in which benefits are expressed 10. What do you mean by number of lives covered in an annuity? Ans. When annuity payments are made with reference to one life or multiple lives is known as number of lives covered. When annuity covers two or more lives especially wife and husband is known as joint and last survivor annuity which continues the same income until the death of the last survivor . Joint life annuity provides a specified income for two or more named persons, with the income ceasing upon the first death among the covered lives. 11. How do you classify annuities based on the method of premium payment? Ans. The method of premium payments in an annuity can be Single payment by beneficiaries Periodic payments 12. How do you classify an annuity based on time when income payments commence? Ans. Annuities can be classified in two ways based on time A deferred annuity An immediate annuity

A deferred annuity, is an annuity purchased with either a single premium or periodic premiums. The first annuity benefit is made after the passage of more than one payment interval. More flexibility is permitted in premium payments with the longer deferral period. An immediate annuity is an annuity purchased with a single premium and the first annuity payment is due [almost] immediately, generally one payment interval [ a month or a year] from the date of purchase. 13. Differentiate Group Life Insurance from Individual Life insurance Ans. Group insurance differs from individual insurance in several respects. GROUP INSURANCE Coverage of many persons under one contract A master contract is formed between the insurer and the group policy owner or the employer INDIVIDUAL INSURANCE Coverage of single person in one Contract A contract is formed between the insurer and individual policy owner

Employees receive a certificate of insurance Individual policy owner holds the that shows they are insured contract It costs less than individual insurance. It costs more than group insurance

Administrative and marketing expenses are Administrative and marketing Reduced as a result of mass distribution expenses are more as a result of methods. individual attention

Individual evidence of insurability is usually Not required Group selection of risks is used

Individual evidence of insurability is necessary. Individual selection of risks is used. Experience rating is used the actual loss experience of the individuals is a major Factor. Individual underwriting rigidity in contract design.

Experience rating is used - the actual loss experience of the group is a major factor.

Group underwriting. Flexibility in contract design Group insurance schemes are in the nature of social security. By recognizing, this the States grant tax advantages to the contracting parties.

14. Explain the eligibility requirements that are commonly required in group insurance plans Ans. The eligibility requirements for group insurance are Eligible groups: Eligible groups are determined by insurance company policy and statutes. Some of the groups which are generally considered as eligible are a. employer - employee groups: Here employer takes a master policy for the benefit of his employees. If the scheme is non contributory i.e., where the employer bears the full cost , all eligible employees must join the scheme. If the scheme is contributory i.e., where the employer bears some part of the cost, a high level of participation by the employees in the scheme is essential. b. Creditor - debtor groups: The master policy is taken out by the creditor to cover the outstanding amount of loans granted to debtors. The master policyholder may be a bank, an employer or an organization.

c. Professional groups: Associations of Doctors, Lawyers, Engineers etc. can be formed on the basis of their profession. d. Cooperative societies: The group should have a reliable identity and should have been formed for a specific purpose other than obtaining Group Insurance benefits. e. Nodal Agencies: State and Central Government departments and welfare organizations are being allowed to take Group Insurance schemes covering some specific groups of weaker sections of the society such as landless agricultural labourers, handloom weavers, taxi drivers and other labour groups. Eligibility requirements: 1. Be full time employees 2. Satisfy a probationary period 3. Apply for insurance during the eligibility period 4. Be actively at work

15. What are the major forms of group life insurance? ANS. The major forms of group life insurance plans are Group term life insurance Group Gratuity scheme Group superannuation scheme 16. Describe the basic characteristics of group term life insurance. Ans. Group term life insurance is the most important form of group life insurance. The insurance provided is yearly renewable term insurance [the premium charged covers the risk for one year and it would be renewed at the end of the year at the option of the group policy holder and this is a step rate premium (the premium increases with the increase in the age), which provides low cost protection to the employees during their working careers The premium rates are determined by conditions of employment, working environment and occupational hazards. The amount of term insurance on an employee' s life is typically one to five times the annual salary or earnings. The term insurance remains in force as long as the employee is part of the group.

If the employee quits or is laid off, he has the right to convert the group term insurance to an individual cash value policy within 31 days without evidence of insurability Group term insurance cannot be converted in to an individual term insurance policy. Most group plans allow a modest amount of life insurance to be written on the employee' s spouse and dependent children. The insurance on the spouse' s life can be converted to an individual cashvalue policy Group term life insurance is used by commercial banks and other lending institutions to insure the lives of debtors. It provides the cancellation of any outstanding debt if the borrower dies Disadvantages: This insurance is temporary and terminates when the individual is no longer part of the group. It is expensive for an older worker to convert to an individual policy after retirement

SECTION C CASE STUDIES


Term, whole life, endowment, annuity policies or the combination of policies are available in the market. Suggest suitable policies for the given situations: 1. You are in 30s. You are the eldest son in your family. One brother and sister are dependent on you for their education and marriage. 2. You are in 40s. You just started a new business. Your entire family is dependent on you. Till you settle in your business you cannot create a capital fund for your family maintenance. 3. You purchased a car by taking loan from Auto Finance. You want to retain that car to your family members even in your absence. 4. You want to raise loans on your policy when you are in need. 5. You want to combine insurance plans with some financial goals like childrens education and marriage 6. You know that compounding factor is important for planning retirement. 7. You do not want to take risk with your VRS Funds 8. You want safe and guaranteed returns from your VRS Funds investment. 9. You want fixed returns from your VRS Funds investment.

ANS:
1 &2 : For these situations the suggested best policy is Term Insurance Policy. These plans offer life insurance cover for specific number of years, at least cost. Since entire premium goes towards the cost of insurance, there is only risk cover and no saving element is involved. 3. The best policy for this situation is Mortgage redemption Insurance policy. These plans offer life insurance cover for specific number of years like till the loan is cleared [or on death, outstanding loan is covered] at the least cost. 4. The best policy for this situation is whole life insurance policy. In term policies and in endowment policies you cannot avail loans. 5. The best policy for this situation is Endowment plans or money-back plans. These policies promise not only the policy face amount on the death of the insured during a fixed term of years, but also the full face amount at the end of the term if the insured survives the term. 6. Consider deferred annuity plans. 7,8&9 The best policy is immediate annuity plans or single premium annuity plans

CHAPTER 9 REGULATION OF INVESTMENT FUNDS OF INSURANCE COMPANIES


SECTION A
1. Sources of investment funds for insurers include a. issuance of policies b. issuance of loans c. issuance of shares d. issuance of only stock e. none of the above

2. Cash inflows from investment activities include a. interest income b. capital gains c. amount realized from sale of securities d. dividends e. all the above

3. Life insurance companies generally invest in a. short term investments b. long term investments c. only bonds d. only equity shares e. none of the above

4. Factors which do not influence investment policy of insurer include a. credit rating b. risk c. return d. advertisement e. none of the above

5. Investment outflows generally result due to a. initial investments b. trading losses c. transaction costs d. all the above 6. Trade off in investments imply the correlation between a. risk/ return b. long term/ short term c. liquidity/ profitability d. liquidity / solvency e. all the above 7. The objective of regulation of insurers investments is a. to protect the insurer b. to maintain solvency c. to earn good returns d. to ensure safety of policyholders funds e. none of the above 8. Investment regulations in India are now defined by a. IRDA b. Central government c. State government d. Ministry of Finance e. none of the above 9. The regulations restricting the investment quantum of an insurer in a particular company/ group/ industry is expressed in a. Solvency norms b. Exposure / Prudential norms c. Liquidation norms d. Capital norms e. none of the above 10. General insurance companies prudently should invest in a. high liquid securities b. only in bonds c. short-term securities d. long-term investments e. none of the above

SECTION - A ANSWERS
(1) a (2) e (3) b (4) d (5) d (6) e (7) d (8) a (9) b (10) c

SECTION - B
1. Define the investment activities of a Life insurer. Ans: The aggregate investment activities of any countrys life and health insurance industry are a major source of capital for the national economic growth. The insurers invest in the debt and equity issues of all types of corporations, shopping malls, apartments and other real estate. They are the major purchases of Government Securities. The total investment portfolio of a life insurance company can be defined on the basis of liabilities that they support. 2. How are assets classified for an insurer? Assets are classified according to the nature of the liabilities for which the assets are held and invested. Assets used to support contractual obligations for guaranteed, fixed benefit payments normally are held in general account while, assets held to support other liabilities associated with investment risk are held in special accounts called separate accounts e.g. (Valuable annuities, pension products, variable life insurance etc). 3. Why is the performance of investment portfolio so critically important for an insurer? Ans: The investment performance of the general account of a life company affects profits, dividends, interest credits on term etc. Separate account performance affects variable life and annuity products and pension funds when benefits by contract depend upon investment results that are passed directly to contract holders. The prices of insurance products are dependent on the investment returns an insurer earns especially the unit linked insurance products. Insurers that earn above average returns can price products favorably to retain customers in a competitive market. Thus, investment management is a process involving asset liability management, integration of investment management and product design management. Hence the performance is so critically essential for the sustainability of an insurer.

4. What are the factors that influence and constrain the investment policy of insurer? Ans: The important economic constraints that influence or act on the investment policy of an insurer include inflation, monetary and fiscal policy of government investment market opportunities, insurers market share, competition, tax liabilities etc. The Regulatory Constraints, rating agencies views, brokers, agents, currency rates, trade balances, geographic location of liabilities are also to be considered in setting investment policy by international insurers. Further, the investment policy of life insurer establishes a level of risk tolerance for individual assets as well as for mismatched assets and liabilities. Hence, the primary objective of an investment policy should be to create an investment portfolio with cash flows, that matches an insurers expected liability cash flows and asset liability risk management strategy, and striking a balance between solvency and profitability. 5. How can you strike a trade off in investment decisions? Ans: Trade off in investment decisions necessarily involves understanding the central concept of investment risk and to acknowledge the relationship of investment risk and expected return. Greater the degree of risk associated with an investment imply greater expected returns. The objective of investment portfolio management is to maximize investment return for a given level of risk or minimize risk for a desired level of return. Investment risk can be defined as the potential variability of returns. The trade off in two areas requires special portfolio management consideration than other investors. The insurance pricing (actuarial) risk is one special area, which takes into account the rate of return to calculate premium. It is also used to discount the price of insurance to policy owners. The discounting practice has two important implications namely: The asset-liability matching (systematic) risk is the second consideration special to financial intermediaries. The changing market interest rate can affect the value of companys assets and liabilities and also the behavior of its customers. Like other investors, life insurers also prefer higher returns to minimize the cost of insurance products and to maximize profitability, keeping in mind the idea of investment margins or spreads.

6. Critically examine the rationale of government intervention in insurance markets. Ans: Governments worldwide intervene in insurance markets either directly or indirectly through regulation of operations or taxation of income. Whatever may be the nature and degree of intervention the basic purpose is to protect the consumers, to raise revenue to support social programs and to ensure order and efficiency in the functioning of insurance markets. Insurance regulation is universal but different approaches are followed worldwide. The main focus of the State is always to ensure safety and solvency of the insurer. Competition is encouraged but is always under check in the areas of policy content and in pricing premiums, where a semblance of uniformity is desired by the policyholders. Insurers are regulated for several reasons, including the following: To maintain insurer solvency, To compensate for inadequate consumer knowledge, To ensure reasonable rates and To make insurance available to those who need it.

7. Identify the main areas of insurance operations that are regulated in India. Ans: Governmental intervention is inevitable in the general interest of the public. The laws that are in place to regulate the operations of the insurers invariably concern the following areas: - formation of insurance companies; - licensing of agents and brokers; - financial requirements for maintaining solvency ; - insurance rates; - investments - sales and claims practices; - taxation and - rehabilitation or liquidation of insurers. Thus it is clear that regulation of insurance companies is in the interests of the public.

SECTION C CASE STUDIES


NIL

CHAPTER 10 LAWS GOVERNING INSURANCE MANAGEMENT


AND ACCOUNTING SECTION - A
1. The LIC was nationalized in the year a. 1955 b. 1962 c. 1956 d. 1966 e. none of the above

2. The GIC was nationalized in the year a. 1977 b. 1947 c. 1972 d. 1975 e. none of the above 3. The Controller of insurance is now replaced by the a. IRDA b. IIRM c. IBRD d. SEBI e. none of the above 4. Which of the following is not the function of the IRDA? a. to regulate and promote orderly growth of the insurance market b. to register insurance companies c. to invest in the capital of the insurance company d. to supervise the functioning of the TAC e. none of the above

5. Which of the following sources of funds is not credited into the IRDA fund? a. government grants, fees, charges b. funds form central government c. percentage of prescribed income from the insurer d. donations from charitable and non-governmental institutions e. all of the above 6. The IRDA fund cannot be utilized for the a. salaries of members and employees b. any other expenses related to the functions of the authority c. allowances and perks of the employees and members d. advertisement of private insurance company e. all of the above 7. Capital adequacy norms of an insurer specify the a. maximum quantum of capital b. minimum paid up equity capital c. minimum reserve capital d. maximum reserve capital e. none of the above 8. Solvency margins refers to the adequate a. excess of assets over liabilities b. excess of over liabilities over assets c. excess of liabilities over cash d. total of assets and liabilities e. none of the above 9. The person engaged in the forecasting and pricing of insurance products is known as a. auditor b. actuary c. advocate d. consultant e. none of the above 10. As per IRDA regulations the registration of an insurer may be suspended if he a. indulges in manipulative insurance business b. fails to provide periodical returns c. fails to make investments as per norms d. fails to furnish information as required by the regulator e. all the above

11. The IRDA has made it obligatory for the insurance companies to procure business from a. social sector b. urban sector c. tertiary sector d. rural sector e. none of the above 12. Assignment provisions refers to a. The transfer of policies b. Purchase of policies c. Buying and selling of policies d. Distribution of policies 13. Absolute assignment implies a. Complete transfer of all rights b. Transfer of some rights c. No transfer of any rights d. Transfer of only obligations 14. Collateral assignment implies a. Temporary / conditional transfer b. Absolute transfer c. Transfer of only obligations d. Transfer of all rights SECTION A ANSWERS (1) c (2) c (3) a (4) c (5) d (6) d (7)b (8) a (9)b (10)e (11) a (12) a (13) a (14) a SECTION - B 1. Enumerate the circumstances when a member can be removed from the office of IRDA? Ans: A member can be removed from office by the Central Government under the following circumstances: 1. 2. 3. When he is adjudged an insolvent When he has become physically or mentally incapable When he has been convicted of any offence which involves moral turpitude

4.

When he has acquired financial interest which is likely to prejudicially affect his function as a member

5. When he has abused his position so as to render his continuation in office detrimental to the public interest 2. Outline the functions of IRDA, which highlights its developmental role? Ans: Out of the innumerable functions of the IRDA, those that highlight its developmental role for the growth of the insurance markets in India are as follows: To regulate, promote and ensure orderly growth of the insurance business and reinsurance business. To protect the interests of the policyholders in matters concerning assigning of policy, nomination, insurable interest, settlement of insurance claim, surrender value of policy, other terms and conditions of contracts of insurance. To promote efficiency in the conduct of insurance business. To call for information form, undertake inspection of, conducting enquires and investigations, including audit of the insurers, intermediaries, insurance intermediaries and other organizations connected with the insurance business. To regulate investment of funds by insurance companies To regulate maintenance of margin for solvency. To settle disputes between insurers and intermediaries or insurance intermediaries. To specify the percentage of life insurance business and general insurance business to be under written by the insurer in the rural or the social sector.

3. Define reinsurance and outline some of the functions of reinsurance? Ans: Reinsurance is a contract of insurance between a primary insurer and a reinsurer where by the primary insurer transfers all or a part of its business to the reinsurer in return for a commission called as the ceding commission. Further, with a good reinsurance arrangement, the primary insurer can: 1. 2. 3. 4. Maximise the retention within the country Develop adequate capacity Secure the best possible protection for the reinsurance costs incurred Simplify the administration of business

The reinsurance programme commences from the beginning of every financial year. If an insurer wants to write inward reinsurance business, it must have a well-defined underwriting policy. SECTION C

CASE STUDIES
1. The insurance regulator will work towards creating an environment that generates confidence among potential policyholders. Why is there a need for creating such an environment? What measures has the IRDA taken in this regard? Ans: Till recently, government owned companies offered insurance services in India and hence there was little doubt in the minds of the policyholders about the safety of their savings. However, the same level of confidence cannot be expected in the case of private companies. To gain this confidence, it is essential that there will be a fairly high level of transparency in their operations. Further, as the risk involved is high, there should be adequate capital base for companies offering these services. Larger capital base is also essential to spread the business and increase the volume of business in order to gain the economies of scale. In this regard, IRDA has issued the following guidelines: Minimum paid-up capital of Rs.100 crore, for life/general insurance and Rs.200 crore, in case of reinsurance business. Deposit with RBI, the least of (a) Rs.10 crore, in cash or marketable approved securities, or (b) a sum of 1% (for life insurance), 3% (for general insurance) of the total gross premium written in India. In case of reinsurer the deposit amount is Rs.20 crore. Annual actuarial investigation. Solvency margins. 2. What strategies do you recommend to a new private sector insurer? Ans: Insurance is comparatively a new business to the Indian corporate world as the LIC and GIC and is subsidiaries were the only players in this market. In this new market, the following strategic issues are to be identified: Tap potential market: There is vast untapped market for insurance in India Investments in secured assets: For an insurance company the risk arises from its liabilities side i.e., due to the policies underwritten.

Thus, investing in secured assets should reduce risks on the assets side. Customized products: A large untapped market also gives scope to customize the products. There was no proper product innovation that has been taking place in the country. Market research: To identify the market to research identity the needs of customer has to be undertaken. While LIC has more than 100 types of policies the level of awareness is very low. Historical data will enable the new players identify the depth in the market and the scope for developing various products. 3. Why are most of the Indian companies venturing into the insurance sector going through the JVs route? Ans: there are three important reasons due to which most of the Indian companies are going through the JVs route to enter into the insurance business: Regulatory prescriptions: Regulations require the Indian players to open an insurance firm in collaboration with a foreign player, which has an equity holding of not more than 26 percent in the company New Business: Insurance is a new type of business and there is no proper experience for the private players in this business. On the other hand, worldwide the insurance market is well developed and there are many companies operating on a global level. In order to gain their expertise, JVs with such players becomes essential. Funds: Insurance is a business, which requires a large capital base, due to the risks involved. Independent start-ups of insurance business by Indian companies alone, will thus not be a feasible option. Instead, when the regulations permit it is better for the Indian players to have a foreign partner who can bring in capital as well as expertise.

CHAPTER- 11 ASSET AND LIABILITY MANAGEMENT


SECTION - A
1. Liquidity risk arises due to a. Mismatch in maturities of assets and liabilities b. Non-withdrawal of deposits on the due date c. Default in repayments of loans d. Both (a) and (b) above e. All the above 2. Which of the following may be present in a loan asset a. credit risk b. interest rate risk c. liquidity risk d. Both (a) and (b) above e. All the above 3. With the convergence of financial institutions, insurance companies are facing the challenge of a. asset management b. liability management c. financial management d. investment management e. all the above 4. The risk of inadequate pricing because of option embedded in the contract of insurance is called as a. asset risk b. pricing risk c. liquidity risk d. actuarial risk e. none of the above

5. Risks which cannot be totally be eliminated by the insurer is called as a. market risk b. credit risk c. systematic risk d. operational risk e. All the above 6. Which of the following statements explain the role of ALM? a. it involves strategic management of balance sheet b. it involves managing risks due to rate fluctuations c. it involves managing credit risk and contingent risks d. Both (a) and (b) above e. All the above 7. Capital management aims to balance the interests of a. owners and management b. owners and policyholders c. owners and regulators d. managers and customers e. none of the above

8. The changes in the asset and liability values due to changes in the interest rates will not be reflected in the books of accounts if the insurer uses the a. market value basis b. book value basis c. replacement value d. reinstatement value basis e. all of the above 9. capital requirement as per the risk profile and size of the assets is called as a. risk based capital (RBC) b. market based capital c. target capital d. growth capital e. none of the above 10. Investments that are neither approved securities or approved investments are called as a. mandated investments b. non- mandated investments c. risky investments d. risk-free bonds e. none of the above

11. Rates which reflect the incidence of death by age are called as a. mortality rates b. morbidity rates c. annuity rates d. premium rates e. none of the above 12. Rates, which reflect the incidence of sickness by age, are called as a. mortality rates b. morbidity rates c. annuity rates d. premium rates e. none of the above 13. reserves maintained by general insurance companies for claims/ losses incurred but nor reported on the balance sheet date are called as a. IBNR claims b. Out-standing claims c. Contingent claims d. Bad claims e. None of the above

(1) a (2) e (3) c (4) d (5) c (6) e (7) a (8) b (9) a (10) b (11) a (12) b (13) a

SECTION A ANSWERS SECTION - B

1. What is meaning of a financial intermediary? What is the speciality of these FIs? Ans: A Financial intermediary (FI) is a firm that brings together users and providers of funds. In the absence of FIs, households generating excess savings by consuming less than their income would have the basic choice of either holding cash as an asset or invest in the securities issued by corporations directly. Financial intermediaries help channelize household savings to the corporate sector. Even, small households often prefer to hold financial claims issued by FIs rather than those issued by corporations.

2. Enumerate the risks faced by insurance companies as financial intermediaries? Ans: Insurers assume various types of risks in providing their services to the public. Insurers identify and manage risks according to a classification of risks developed by the actuarial profession. Insurance companies which is basically a financial intermediary face some unique risks such as: Asset risk. Pricing risk Interest rate risk Market / Systematic risk Credit risk Off-balance sheet risk Technology risk Operational risk

All the above risks are interrelated. The effective management of these risks is central to the performance of an insurance company as an FI. Asset Risk: Asset risk reflects the riskiness of the asset portfolio of the life insurer. Asset risk or Asset Depreciation risk refers to the risk of decline or decrease in the value of assets due to: default by borrowers decline in the market value of investments due to fluctuations in interest rate. Asset risk reduces the capital. It can be averted by careful financial management through credit and investment analysis, using readily available asset market values. To calculate the quantum of adequate capital requirement, a credit risk weight is multiplied by the face value of the asset on the balance sheet. Pricing Risk Pricing Risk or Pricing inadequacy risk refers to the risk that the value of insurers liabilities may exceed value of their assets. This may be due to Inadequate pricing of the policy. Higher mortality/morbidity rate

Higher Policy lapses Increase in expenses than anticipated This insurance risk captures the risk of adverse changes in mortality risk and Morbidity risk. Mortality risk refers to the risk of death, while the morbidity risk refers to the risk of ill health. Although, with the help of mortality tables, Insurers, have accurate idea of the insureds timing of death, sometimesunexpected contingencies, such as AIDS, can upset these predictions. As a result, insurers adjust insurance in force for the current level of reserves and multiply the resulting number by an insurance risk factor. Similar calculations are carried out for accident and health insurance, which covers morbidity (ill health risk). Hence, to avoid pricing risk, during product designing and pricing, the above factors and assumptions influencing the pricing decisions should be carefully considered or it may affect the liquidity position of the insurers. Interest-rate risk: In mismatching their assets and liabilities, as a part of their asset transformation function, FIs expose themselves to interest-rate risk. The asset transformation function involves buying primary securities and issuing secondary securities. The primary securities purchased by FIs have maturity and liquidity characteristics different from those of secondary securities exposing FIs to interest-rate risk. Whenever an FI holds longer-term assets relative to liabilities, it potentially exposes itself to refinancing risks. Similarly, whenever an FI holds shorter-term assets relative to its liabilities, it is exposed to reinvestment risk. This is because of uncertainty about the interest rate at which it could reinvest funds borrowed for a longer period. Market risk: FIs face market risk when they actively trade their assets and liabilities, instead of holding them for longer-term investment, funding or hedging purposes. Market risk occurs, when prices change in a direction opposite to that expected. As a result, the more volatile are asset prices, the greater are the market risks faced by FIs, and so they need to measure market risk on a day-to-day basis. Credit risk: Credit risk arises, when promised cash flows on the primary securities held by FIs are not paid in full. The FIs face no credit risk, when all the financial claims held by the FIs are paid in full on maturity and interest payments are made on promised dates. Similarly, if a borrower defaults, the principal loaned and the interest payments expected to be received are at risk. Off-balance sheet risk: With the growth in their off-balance sheet activities, the FIs are increasingly being exposed to off-balance sheet risks. These activities

include those related to contingent assets and liabilities, not shown in the balance-sheet, but affect the future of the FI (e.g. letters of credit issued, loan commitments by banks, mortgaging servicing contracts, positioning in Futures, Forwards, Swaps, and other derivatives). Technological and operational risks: These risks have been a major concern of FIs in recent years. All major FIs focus on improving their operational efficiency with major investments in internal and external communications, computers, technological infrastructure.[ Automated teller machine(ATM), Automated clearing houses (ACH), Clearing house interbank payment system (CHIPS)]. These efforts aim at improving performance, increasing profits, lowering operational costs, and capturing new markets for the FI. Technological risk occurs when technological investments do not produce the anticipated cost savings. Operational risk occurs whenever existing technology malfunctions or backoffice systems break down. Foreign exchange risk: Global FIs have the advantage and the potential to expand their operations and investment activities abroad directly or to expand a financial asset portfolio so as to include foreign securities as well as domestic securities. Foreign exchange risk is the risk that exchange rate changes can affect the value of an FIs assets and liabilities located abroad. Country or Sovereign risk: FIs face a much more serious credit risk, in case of country or sovereign risk. It is the risk that repayments from foreign borrowers may be interrupted because of interference from foreign governments. Liquidity risk: Liquidity risk arises whenever an FIs liability holders, such as depositors, policyholders demand immediate cash for their financial claims. This situation may force the FIs either to borrow additional funds or sell off their assets in a very short period of time and at low prices to meet the demand for withdrawal of funds. Insolvency risk: Insolvency occurs whenever the internal capital or equity resources of an FIs owners are inadequate to meet losses incurred due to one or more risks. Insolvency risk is a consequence or outcome of excessive interest rate, market, credit, off-balance sheet, and liquidity risks. Other risks: These are often called event risks. Involving sudden and unexpected changes in financial market conditions due to war, revolution, or sudden

collapse in stock markets, fraud, theft, malfeasance, breach of trust, and other macroeconomic risks such as inflation, unemployment, 3. Can insurance reduce their risk by diversification ? Ans: Diversification reduces risk, because prices of different securities do not move exactly together. But, insurers face two types of risk. Unique Risk Systematic or Market Risk Unique risk is derived from the uniqueness of each firm, from its unique environment. The unique risk of each stock is countered by unique risk of other stocks thus reducing overall risk. Systematic or market risk is the unavoidable risk that stems from factors which influence all businesses alike such as societal influences and factors. The number of securities determine the risk profile. If a portfolio consists of a few securities, the firms face greater unique risk and diversification of portfolio eliminates a greater part of unique risk. The minimum rate of return required by investors to hold a security depends upon the opportunity cost of funds i.e. the returns available from alternative asset investments. 4. Discuss ALM as a multidisciplinary exercise Ans: Asset - liability Management (ALM) is a multidisciplinary undertaking. It involves Product contract designing Asset management Products should satisfy consumers wants or expectations as to price, service, performance etc, and prevent book value cash withdrawals leading to liquidity problems. Asset management involves a thorough valuation of its assets to earn maximum return on its invested assets keeping in view the company' s risk posture. Asset-liability management essentially involves managing the Asset portfolio structure Amount of duration mismatch allowed Extent of asset-liability mismatch allowed

Basically assets risk also implies a credit risk of life insurer which in turn includes the risk of default on investments. Therefore the investment policy sets limits on portfolio exposure by industry, geographic region, type of business, individual company. Besides credit risk, another factor important in the ALM exercise is the liquidity factor, which is a major concern for Insurers. To reduce the magnitude and probability of adverse cash flows, insurers, use various techniques to control cash flows such as liability controls Asset controls Liability Controls focus of liability control is to discourage or prevent book value cash withdrawals leading to liquidity problems. These controls are generally included in the contract itself by way of provisions and conditions to create additional cash flows either to policy owners if they persist, or to the Insurer, when withdrawal options are exercised. On the other hand, Asset Controls focus on controls is to ensure a maximum rate of return from an investment portfolio for a given level of profitability and solvency risk. Asset Controls insist on sufficient liquidity to meet their current obligations. 5. What is the need for capital adequacy? The modern FIs today are exposed to numerous risks due to their inherent risky contracts, resulting from both on-and off balance sheet activities. Sometimes these risks could be from their domestic business and also from their international contracts. However, to ensure stability survival and growth in the market, an FI especially an insurer needs to protect against the risk of insolvency, i.e. shield it from failing or from liquidation. The primary means of protection to shield an FI, against the risk of insolvency and failure is an FIs capital. Hence the need to maintain adequate capital is to be emphasized. Thus, capital adequacy reflects the adequate quantum of capital to meet the following functions. To absorb unanticipated losses. To protect uninsured depositors, bond holders, and creditors in the event of insolvency and liquidation. To protect FI insurance funds and the tax payers.

To acquire sufficient investments necessary to provide financial services. 6. What is meant by Capital Management Ans: Insurance executives and managers face two critically important and conflicting objectives. On one hand, they must satisfy the owners of the firm, who are interested in maximizing the return on their equity (ROE), which means that their interest generally lie in minimizing their equity investment (i.e. the firm' s capital) On the other hand, the executives must satisfy their customers (policy owners) and regulators, who are interested in the financial solidity of the firm, which means their interests lie in maximizing the firm's capital. Capital Management is concerned with balancing these interests. The term capital, capital and surplus, networth, and equity all reflect the difference between assets and liabilities. 7. Outline the functions of capital in a company. Ans: Capital within a life insurer serves four functions such: Protect the policy owners in the event of insurer insolvency. Minimize or eliminate the direct costs to other insurers and taxpayers from insolvency assessments. Absorb unanticipated losses and thereby inspire confidence and enable the insurer to continue as a going concern. Provide basis for future growth.

SECTION C CASE STUDIES


Given below is a table that shows the relative asset risk weights for Life and Property Casualty insurers.

Risk Based Capital (RBC) Factors for Selected Assets Assets Bonds Government AAA-A* BBB BB B CCC In or near default Residential mortgages Commercial mortgages Common stock Preferred stock Life insurers Property insurers 0.0% 0.3% 1.0% 4.0% 9.0% 20.0% 30.0% 0.5% 3.0% 30.0% 2.0% 0.0% 0.3% 1.0% 2.0% 4.5% 10.0% 30.0% 5.0% 5.0% 15.0% 2.0%

1. Interpret the given table. 2. How is the risk based capital decided for an insurance company? Ans: 1. RBC or risk based capital requirements have become essential area of concern in the financial management of insurance company. This strategy ensures both solvency and liquidity requirements of these companies. As per the above given information, an insurer with Rs.100 million in common stock would have a riskbased capital requirement of Rs.30 million, while for one with Rs. 100 million in BBB corporate bonds only Rs. 1 million would be required. 2. The following factors are considered for computing Risk Based Capital (RBC): i) ii) Risk characteristics of the company based on its size, nature of business (say life, motor, fire, etc.), quality of investments made, etc. Risk weight-age for each class of assets. For example different weights are applied to investment in government securities, private loans, equities, real estate and other investments.

iii) iv)

Total adjusted capital which consist of statutory capital, asset valuation reserve and any other voluntary reserves Capital derived from risk-weighted formula as per point (ii) above.

The two capitals, viz. as per iii) and iv) are compared to find out whether the capital is adequate in relation to the risk of the company. The regulators take different action depending on the Total Adjusted Capital (TAC) and the Authorised Control level (ACL). For example in US the ACC is determined based on a formula that takes into account the insurance companies risks viz. asset risks, insurance risks, interest risk and business risk. In India the IRDA has prescribed methodology for calculations of solvency margins, which needs to be reported to the regulator in the prescribed forms.

You might also like